a

Menu

M

Chiudi

Massimi e minimi per funzioni in più variabili: una guida essenziale

Massimi e minimi liberi e vincolati, Teoria Funzioni di più variabili

Home » Massimi e minimi per funzioni in più variabili: una guida essenziale

Benvenuti nella nostra guida ai massimi e minimi per funzioni in più variabili.

Nelle applicazioni, una funzione di più variabili spesso rappresenta una grandezza fisica o una quantità che varia in relazione a dei parametri: ad esempio un’energia, una distanza, un costo, un profitto relativi a un particolare contesto. Un obiettivo fondamentale è quello di massimizzare o minimizzare queste quantità, e ciò corrisponde matematicamente a studiare le funzioni che le descrivono, in particolare determinandone massimi e minimi.

Questa dispensa è una guida che spiega i principali strumenti disponibili per effettuare questo studio:

  • Definizione di estremi per funzioni in più variabili;
  • Teorema di Fermat per funzioni in più variabili e classificazione dei punti stazionari in punti di massimo e minimo locale o di sella.
  • Determinazione della natura dei punti stazionari attraverso lo studio della matrice hessiana; condizioni necessarie e sufficienti nei casi in cui la matrice hessiana sia definita positiva o negativa, indefinita o semidefinita, attraverso lo studio del determinante hessiano e il criterio di Sylvester.
  • Strategie utili quando lo studio della matrice hessiana non consente di determinare la natura del punto stazionario in esame.

Il testo è completato dalle raccolte

con ulteriori e interessanti problemi svolti. Attraverso esempi dettagliati ed esercizi mirati, questa guida si rivela un’indispensabile risorsa didattica per chiunque desideri padroneggiare quest’area affascinante della matematica.
Buona lettura!

 
 

Sommario

Leggi...

In questa dispensa vengono definiti e studiati i massimi e minimi di funzioni in più variabili. Si riassume la teoria necessaria per affrontare gli esercizi, ponendo l’attenzione al caso di due e tre variabili. Vengono esplorate anche situazioni in cui le funzioni da studiare non rispettano le ipotesi dei teoremi proposti, in cui i metodi tradizionali falliscono.

 
 

Autori e revisori


 
 

Notazioni

Leggi...

\mathbb{N}    Insieme dei numeri naturali;
\mathbb{Z}    Insieme dei numeri interi;
\mathbb{R}    Insieme dei numeri reali;
\mathbb{R}^+    Insieme dei numeri reali positivi;
\mathbb{R}^-    Insieme dei numeri reali negativi;
\mathbb{R}^n    Spazio euclideo n-dimensionale;
\partial A    Frontiera dell’insieme A;
A^\circ    Parte interna dell’insieme A;
\mathcal{M}_n(\mathbb{R})    Insieme delle matrici quadrate di ordine n con coefficienti reali;
\operatorname{Id}    Matrice identità;
\det(M)    Determinante della matrice M;
\dfrac{\partial f}{\partial x_i}    Derivata parziale i-esima di f;
f_{x_ix_j},\dfrac{\partial^2f}{\partial x_i\partial x_j}    Derivata parziale seconda rispetto alle variabili x_i, x_j;
\nabla^2f    Matrice hessiana di f;
\mathcal{C}^k(A)    Spazio delle funzioni continue su A, con derivate parziali di ordine k continue su A.


 
 

Introduzione

Leggi...

Questa dispensa ha per soggetto lo studio della natura dei punti critici di funzioni di più variabili reali, capitolo fondamentale dei corsi di Analisi II per matematica, fisica ed ingegneria. La teoria sottostante agli esercizi si basa sullo studio della matrice hessiana (analogo multimensionale della derivata seconda) attraverso criteri riguardanti forme quadratiche (associate alla matrice hessiana per via della sua simmetria, [1, §6.3]). Vi è tuttavia un’evidente distinzione da sottolineare da cui dipende la difficoltà dell’esercizio e riguarda la definitezza della matrice hessiana stessa:

    \[\quad\]

  • se la matrice hessiana della funzione valutata in uno dei suoi punti critici è definita o indefinita, l’esercizio può essere considerato “standard” e affrontabile con la teoria che sviluppiamo in queste note (vedi esercizio guida);
  •  

  • se, invece, la matrice hessiana risulta semidefinita, ma non definita, questa teoria non è efficace per concludere la natura di tale punto critico e altre tecniche devono essere utilizzate per conseguire questo obiettivo.

Purtroppo, nella seconda situazione, non vi è un approccio uniforme e devono essere utilizzati metodi da adattare caso per caso. Mostreremo con diversi esempi come affrontare queste situazioni. Esse possono consistere nel provare che il punto è di massimo, minimo o sella utilizzando esplicitamente la definizione (esempi 1.20, 1.19), oppure mediante altri metodi ad-hoc (esempio 1.34). Viene inoltre considerato il caso in cui la funzione non sia sufficientemente differenziabile (esempio 1.9).

L’ultima sezione è infine una raccolta di esercizi in cui si applica quanto espresso precedentemente.


 
 

Teoria in pillole

Introduzione.

In questa sezione fissiamo le definizioni e i concetti principali, rimandando a [1, §4] per approfondimenti.

Definizione 1.1 (punto critico). Sia f:A\rightarrow\mathbb{R} una funzione, con A\subseteq\mathbb{R}^n sottoinsieme aperto. Un punto \overline{x}_0\in A si dice punto critico di f se f possiede derivate parziali in \overline{x}_0 e vale

    \begin{equation*}     \nabla f(\overline{x}_0)\coloneqq \left(\dfrac{\partial f}{\partial x_1}(\overline{x}_0),\dfrac{\partial f}{\partial x_2}(\overline{x}_0),...,\dfrac{\partial f}{\partial x_n}(\overline{x}_0)\right)=(0,...,0). \end{equation*}

    \[\quad\]

Esplicitamente, un punto \overline{x}_0 è punto critico di f se è soluzione del seguente sistema ad n equazioni

    \begin{equation*} \begin{cases} \dfrac{\partial f}{\partial x_1}(\overline{x}_0)=0  \\ \quad \vdots \\ \dfrac{\partial f}{\partial x_n}(\overline{x}_0)=0. \end{cases} \end{equation*}

Esempio 1.2. Sia f:\mathbb{R}^3\rightarrow\mathbb{R}, definita da f(x,y,z)\coloneqq 2xy+xz^2, per ogni (x,y,z)\in\mathbb{R}^3. I punti critici di f soddisfano il seguente sistema di equazioni: Equations and Visualization

    \begin{equation*}                 \left\{\begin{array}{l}                      \dfrac{\partial f}{\partial x}(x,y,z)=2y+z^2=0  \\\\                       \dfrac{\partial f}{\partial y}(x,y,z)=2x=0\\\\                       \dfrac{\partial f}{\partial z}(x,y,z)=2xz=0.                 \end{array}\right.\Leftrightarrow\left\{\begin{array}{l}                       y=-\dfrac{z^2}{2} \\\\                       x=0.                 \end{array}\right.                 \end{equation*}

Dunque l’insieme dei punti critici di f è la parabola \Gamma in \mathbb{R}^3 parametrizzata da

(1)   \begin{equation*}                 \gamma(t)=\left(0,-\dfrac{t^2}{2},t\right)                 \qquad                 \forall t\in\mathbb{R}                 \end{equation*}

e rappresentata in azzurro in figura 1.

Massimi e minimi per funzioni in più variabili

Figura 1: in azzurro la parabola \Gamma, inclusa nel piano yz, costituita dai punti critici della funzione f dell’esempio 1.2.

    \[\quad\]

Definizione 1.3. Sia f:A\rightarrow\mathbb{R} una funzione, con A\subseteq\mathbb{R}^n. Un punto \overline{x}_0\in A si dice:

  • punto di massimo relativo se esiste un intorno aperto \mathcal{U}_{\overline{x}_0} di \overline{x}_0 tale che

        \begin{equation*}         f(\overline{x})\leq f(\overline{x}_0),\qquad\forall\bar{x}\in\mathcal{U}_{\overline{x}_0}\cap A;     \end{equation*}

      

Rendered by QuickLaTeX.com

  • punto di minimo relativo se esiste un intorno aperto \mathcal{U}_{\overline{x}_0} di \overline{x}_0 tale che

        \begin{equation*}         f(\overline{x})\geq f(\overline{x}_0),\qquad\forall\bar{x}\in\mathcal{U}_{\overline{x}_0}\cap A;     \end{equation*}

      

  • Rendered by QuickLaTeX.com

  • punto di sella se \overline{x}_0 è un punto critico di f e per ogni intorno aperto \mathcal{U}_{\overline{x}_0} di \overline{x}_0, esistono \overline{y},\overline{z}\in\mathcal{U}_{\overline{x}_0}\cap A tali che

        \begin{equation*}         f(\overline{y})\leq f(\overline{x}_0)\leq f(\overline{z}),     \end{equation*}

    ossia se \overline{x}_0 non è né punto di massimo, né punto di minimo;  

  • Rendered by QuickLaTeX.com

  • punto di massimo assoluto se

        \begin{equation*}         f(\overline{x})\leq f(\overline{x}_0),\qquad\forall\bar{x}\in A;     \end{equation*}

  •  

  • punto di minimo assoluto se

        \begin{equation*}         f(\overline{x})\geq f(\overline{x}_0),\qquad\forall\bar{x}\in A.     \end{equation*}

  • Infine, f(\overline{x}_0)\in\mathbb{R} si dice massimo relativo (risp. assoluto) se \overline{x}_0 è un punto di massimo relativo (risp. assoluto); f(\overline{x}_0)\in\mathbb{R} si dice minimo relativo (risp. assoluto) se \overline{x}_0 è un punto di minimo relativo (risp. assoluto);

        \[\quad\]

    Osservazione 1.4. È chiaro che se \overline{x}_0 è un punto di massimo (o minimo) assoluto è, in particolare, un punto di massimo (o minimo) relativo.

    Esempio 1.5. Consideriamo la seguente funzione f:\mathbb{R}^2\rightarrow\mathbb{R}, definita da

        \begin{equation*}     f(x,y):=x^3-y^3-3x+12y,\qquad \forall (x,y)\in\mathbb{R}^2. \end{equation*}

    Verifichiamo se f ammette massimo o minimo assoluti.

    Svolgimento. Condizione necessaria affinché f ammetta massimo o minimo assoluti è che f(\mathbb{R}^2) sia un sottoinsieme superiormente o inferiormente limitato di \mathbb{R}. Tuttavia, presenteremo due direzioni lungo le quali la funzione assume valori in modulo sempre maggiori, tendenti ad infinito, contraddicendo la limitatezza di f(\mathbb{R}^2).

    Consideriamo, ad esempio, la semiretta \Gamma_+ parametrizzata da

        \begin{equation*}     \Gamma_+:=\{(t,0)\mid t\in\mathbb{R}^+\} \end{equation*}

    e rappresentata in verde in figura 2. La restrizione f\big|_{\Gamma_+} di f a \Gamma_+ risulta essere definita da

        \begin{equation*}     f(t,0)=t^3-3t,\qquad \forall t \geq 0 \end{equation*}

    e allora

        \begin{equation*}     \lim_{t\to+\infty}f(t,0)=+\infty. \end{equation*}

    Questo permette di concludere che f non ammette massimo assoluto, poiché f(\mathbb{R}^2) è superiormente illimitato.

    Proviamo infine che f non ammette nemmeno minimo. Consideriamo la semiretta

        \begin{equation*}     \Gamma_-:=\{(t,0)\mid t\in\mathbb{R}^-\}=\{(-t,0)\mid (t,0)\in\Gamma_+\}, \end{equation*}

    rappresentata in rosso in figura 2. Dato che la restrizione di f all’asse x è una funzione dispari, risulta

        \begin{equation*}     \lim_{t\to-\infty}f(t,0)=-\lim_{t\to-\infty}f(-t,0)=-\lim_{t\to+\infty}f(t,0)=-\infty. \end{equation*}

    Ciò prova che f(\mathbb{R}^2) è inferiormente illimitato e dunque f non ammette nemmeno minimo assoluto. In questo esempio viene enfatizzato separatamente che la funzione presenta cammini lungo cui è inferiormente e superiormente illimitata. Più sinteticamente, bastava osservare che la restrizione di f all’asse x è illimitata sia superiormente, sia inferiormente, in particolare \sup f = + \infty e \inf f=-\infty, cioè f non ammette massimi e minimi assoluti.

        \[\quad\]

        \[\quad\]

    Massimi e minimi per funzioni in più variabili

    Figura 2: le semirette \Gamma_+ e \Gamma_- dell’esempio 1.6. La restrizione di f a queste semirette è illimitata.

        \[\quad\]

        \[\quad\]

    Esempio 1.6. Consideriamo la seguente funzione f:\mathbb{R}^2\rightarrow\mathbb{R}, definita da

        \begin{equation*}     f(x,y):=x^2+y^2,\qquad \forall (x,y)\in\mathbb{R}^2. \end{equation*}

    Verifichiamo se f ammette massimo o minimo assoluti.

    Svolgimento. Osserviamo immediatamente che f non è superiormente limitata, infatti, procedendo come nell’esempio precedente, consideriamo la retta \Gamma\coloneqq\{(t,0)\mid t\in\mathbb{R}\}; la restrizione di f a \Gamma è

        \begin{equation*} f(t,0)=t^2\qquad\forall t\in\mathbb{R}, \end{equation*}

    da cui

        \begin{equation*} \lim_{t\to\pm\infty}f(t,0)=+\infty. \end{equation*}

    Mostriamo tuttavia che 0=f(0,0) è minimo assoluto di f. Ciò segue immediatamente dal fatto che

        \begin{equation*} f(x,y)=x^2+y^2\geq 0\qquad\forall (x,y)\in\mathbb{R}^2. \end{equation*}

    Dunque f ammette minimo assoluto pari a 0, ma non ammette massimo assoluto.


    Teoremi del calcolo differenziale.

    Presentiamo in questa sezione alcuni dei teoremi centrali riguardo lo studio di massimi e minimi attraverso l’analisi dei punti critici.

    Il seguente teorema è fondamentale nello studio di problemi di massimo e minimo. Esso fornisce una condizione necessaria affinché una funzione differenziabile assuma massimo o minimo relativo in un punto, nonché una relazione tra punti di massimo (e minimo) e punti critici.

    Teorema 1.7 (Fermat). Sia f:A\rightarrow\mathbb{R}, con A\subseteq\mathbb{R}^n. Supponiamo che

        \[\quad\]

    1. \overline{x}_0 sia un punto di massimo (o minimo) relativo di f;
    2.  

    3. \overline{x}_0\in A^\circ;
    4.  

    5. esistono le derivate parziali di f in \overline{x}_0.

    Allora \overline{x}_0 è un punto critico di f, ovvero vale

        \begin{equation*}     \nabla f(\overline{x}_0)=(0,...,0). \end{equation*}

        \[\quad\]

    Dimostrazione. Vedi [1, teorema 3.17].

    Questo teorema stabilisce che nella ricerca dei punti di massimo e di minimo relativi di f sia sufficiente considerare solamente i suoi punti critici o i punti in cui le derivate parziali di f non esistono o, infine, i punti di frontiera del dominio. Ciò significa che è possibile scartare tutti i punti \overline{x} interni al dominio, in cui le derivate parziali di f esistono e \nabla f(\overline{x})\neq(0,...,0).

    Osservazione 1.8. Sia la condizione \overline{x}_0\in A^\circ, sia l’esistenza delle derivate parziali di f in \overline{x}_0 sono necessarie per la validità del teorema di Fermat 1.7. Possiamo infatti considerare i seguenti controesempi in una dimensione, in cui le derivate parziali sono sostituite dall’unica derivata in senso ordinario:

    • Punto non interno: consideriamo A = [0,1] e la funzione f \colon [0,1] \to \mathbb{R}, definita da

          \[f(x)=x,\qquad \forall x\in[0,1].\]

      \overline{x}_0=1 è un punto di massimo relativo per f e f è differenziabile in \overline{x}_0, tuttavia \nabla f(\overline{x}_0)=f'(1)=1\neq 0. Tuttavia, il teorema di Fermat 1.7 non si applica poiché 1\in\partial A=A\setminus A^\circ. Lo stesso vale per il punto di minimo per f, \overline{x}_1=0, in cui il gradiente non si annulla, ma non sono rispettate le ipotesi del teorema, essendo \overline{x}_1 un punto di frontiera di A.

    Massimi e minimi per funzioni in più variabili
  • Punto di non differenziabilità: consideriamo A = [-1,1] e la funzione f \colon [-1,1] \to \mathbb{R}, definita da

        \[f(x)=|x|,\qquad \forall x\in[-1,1].\]

    \overline{x}_0=0 è un punto di minimo relativo per f e 0\in A^\circ, tuttavia non esiste \nabla f(\overline{x}_0), in particolare, non possiamo concludere che \nabla f(\overline{x}_0)=0. In questo caso il teorema di Fermat 1.7 non si applica poiché non esiste la derivata di f in \overline{x}_0.

  • Massimi e minimi per funzioni in più variabili

    In generale, non è detto che le derivate parziali di una funzione esistano in ciascun punto del suo dominio. Occorre quindi studiare la natura di tali punti utilizzando altri metodi, che richiedono informazioni particolari sulla funzione in esame. Il prossimo esempio mostra come si possa procedere in uno di questi casi.

    Esempio 1.9. Studiamo i punti stazionari e i punti di massimo o minimo locale di f:\mathbb{R}^2\rightarrow\mathbb{R}, definita da

        \begin{equation*}     f(x,y):=\sqrt{4x^2+y^2}, \qquad\forall (x,y)\in\mathbb{R}^2. \end{equation*}

    Svolgimento. Mostriamo che f non è derivabile in (0,0). Ricordiamo che f è derivabile in (0,0) se e solo se esistono finite le derivate parziali in (0,0). Proviamo a calcolare la derivata di f rispetto ad x applicando la definizione:

        \begin{equation*}     \dfrac{\partial f}{\partial x}(0,0):=\lim_{h\to 0^\pm}\dfrac{f(h,0)-f(0,0)}{h}=\lim_{h\to0^\pm}\dfrac{\sqrt{4h^2}}{h}=\lim_{h\to0^\pm}\dfrac{2|h|}{h}=\pm1. \end{equation*}

    Poiché limite destro e limite sinistro non coincidono, non esiste la derivata parziale di f rispetto ad x e quindi f non è derivabile in (0,0). Concludiamo anche che (0,0) non è un punto critico di f, poiché per definizione un punto critico è un punto di derivabilità in cui si annulla il gradiente.

    Ricerchiamo esplicitamente i punti critici di f, calcolando il gradiente di f attraverso le derivate parziali laddove esse esistono, ovvero su \mathbb{R}^2\setminus\{(0,0)\}. Assumiamo dunque (x,y)\neq(0,0) e otteniamo

        \begin{equation*}     \dfrac{\partial f}{\partial x}(x,y)=\dfrac{8x}{2\sqrt{4x^2+y^2}}=\dfrac{4x}{\sqrt{4x^2+y^2}} \end{equation*}

        \begin{equation*}     \dfrac{\partial f}{\partial y}(x,y)=\dfrac{2y}{2\sqrt{4x^2+y^2}}=\dfrac{y}{\sqrt{4x^2+y^2}}, \end{equation*}

    da cui

    (2)   \begin{equation*}     \nabla f(x,y)=\left(\dfrac{4x}{\sqrt{4x^2+y^2}},\dfrac{y}{\sqrt{4x^2+y^2}}\right)\qquad \forall(x,y)\neq(0,0). \end{equation*}

    I punti critici di f sono allora le coppie (x,y)\in\mathbb{R}^2\setminus\{(0,0\} che annullano il gradiente, ovvero soddisfano

        \begin{equation*}     \nabla f(x,y)=\left(\dfrac{4x}{\sqrt{4x^2+y^2}},\dfrac{y}{\sqrt{4x^2+y^2}}\right)=(0,0). \end{equation*}

    Risolviamo pertanto il sistema

        \begin{equation*}     \left\{\begin{array}{l}          \dfrac{4x}{\sqrt{4x^2+y^2}}=0\\\\          \dfrac{y}{\sqrt{4x^2+y^2}}=0.      \end{array}\right. \end{equation*}

    Ricordiamo che abbiamo supposto (x,y)\neq(0,0), quindi \sqrt{4x^2+y^2}\neq0 e possiamo moltiplicare entrambi i membri di entrambe le equazioni per la quantità non nulla \sqrt{4x^2+y^2}, ottenendo in questo modo

        \begin{equation*}     \left\{\begin{array}{l}          4x=0\\\\          y=0     \end{array}\right.\Leftrightarrow     \left\{\begin{array}{l}          x=0\\\\          y=0.      \end{array}\right. \end{equation*}

    L’unica soluzione del sistema è il punto (x,y)=(0,0) che tuttavia non è accettabile, poiché l’equazione (2) vale solo per (x,y)\neq(0,0). Concludiamo allora che f non ha punti critici.

    Il fatto che f non abbia punti critici non significa necessariamente che non presenti punti di massimo o minimo locale. Infatti, possiamo applicare il teorema di Fermat 1.7 solo sull’insieme aperto \mathbb{R}^2\setminus\{(0,0)\}, su cui f è differenziabile: poiché nessun punto soddisfa la condizione necessaria imposta dal teorema di Fermat, concludiamo che (x,y) non è punto di massimo o di minimo locale per f, qualsiasi sia (x,y)\in\mathbb{R}^2\setminus\{(0,0)\}.

    Rimane, tuttavia, da indagare il punto di non derivabilità (0,0), su cui il criterio di Fermat non può essere applicato. Siamo costretti ad utilizzare la definizione di punto di massimo o di minimo relativo. Osserviamo infatti che f è una funzione non negativa, ovvero f(\mathbb{R}^2)\subseteq[0,+\infty). Ciò significa che

        \begin{equation*}     0=f(0,0)\leq f(x,y)\qquad\forall (x,y)\in\mathbb{R}^2. \end{equation*}

    Dunque, per definizione, (0,0) è un punto di minimo assoluto per f e, in particolare, è anche un punto di minimo relativo per f. Il grafico di f in un intorno dell’origine è rappresentato in figura 3.

        \[\quad\]

        \[\quad\]

    Massimi e minimi per funzioni in più variabili

    Figura 3: grafico della funzione f dell’esempio 1.9. Si vede che f non è differenziabile nell’origine, ma tale punto è di minimo assoluto.

        \[\quad\]

        \[\quad\]

    Nei punti di regolarità, il prossimo strumento si rivelerà fondamentale.

    Definizione 1.10 (matrice hessiana). Sia A\subseteq\mathbb{R}^n un insieme aperto, f:A\rightarrow\mathbb{R} una funzione e sia \overline{x}\in A. Supponiamo che in \overline{x} esistano tutte le derivate seconde parziali di f, \dfrac{\partial^2 f}{\partial x_i\partial x_j}(\overline{x}), per i,j=1,...,n. Si definisce matrice hessiana di f in \overline{x} la matrice contenente tutte le derivate parziali seconde di f in \overline{x}:

        \begin{equation*}     \nabla^2f(\overline{x})\coloneqq     \begin{pmatrix}     \dfrac{\partial^2f}{\partial x_1^2}(\overline{x})&\dfrac{\partial^2f}{\partial x_1\partial x_2}(\overline{x})&\cdot\cdot\cdot&\dfrac{\partial^2f}{\partial x_1\partial x_n}(\overline{x})\\[5pt]     \dfrac{\partial^2f}{\partial x_2\partial x_1}(\overline{x})&\dfrac{\partial^2f}{\partial x_2^2}(\overline{x})&\cdots&\dfrac{\partial^2f}{\partial x_2\partial x_n}(\overline{x})\\[5pt]     \vdots&\vdots&\ddots&\vdots\\[5pt]     \dfrac{\partial^2f}{\partial x_n\partial x_1}(\overline{x})&\dfrac{\partial^2f}{\partial x_n\partial x_2}(\overline{x})&\cdot\cdot\cdot&\dfrac{\partial^2f}{\partial x_n^2}(\overline{x})     \end{pmatrix}. \end{equation*}

        \[\quad\]

    Il seguente teorema afferma che, sotto opportune ipotesi di regolarità, la matrice hessiana è sempre simmetrica.

    Teorema 1.11 (Schwarz). Sia A \subseteq \mathbb{R}^n un insieme aperto e sia f \colon A \to \mathbb{R} una funzione di classe \mathcal{C}^2(A)1. Allora per ogni i,j=1,...,n e per ogni \overline{x}\in A,

        \[\dfrac{\partial^2 f}{\partial x_i\partial x_j}(\overline{x})=\dfrac{\partial^2 f}{\partial x_j\partial x_i}(\overline{x}).\]

    In altre parole, la matrice hessiana \nabla^2f è simmetrica.    


    1. Una funzione f si dice di classe \mathcal{C}^2(A) su un insieme aperto A se la funzione stessa e le sue derivate di primo e secondo ordine sono funzioni continue su A.

        \[\quad\]

    Dimostrazione. Vedi [1, teorema 3.14].

    Esempio 1.12. Sia f:\mathbb{R}^3\rightarrow\mathbb{R}, definita da f(x,y,z)\coloneqq x^3+xy^2z, per ogni (x,y,z)\in\mathbb{R}^3. Calcoliamo la matrice hessiana di f.

    Svolgimento. Calcoliamo le derivate prime parziali di f:

        \begin{equation*} \begin{split}         \dfrac{\partial f}{\partial x}(x,y,z)&=3x^2+y^2z,\qquad\forall(x,y,z)\in\mathbb{R}^3;\\         \dfrac{\partial f}{\partial y}(x,y,z)&=2xyz,\qquad\forall(x,y,z)\in\mathbb{R}^3;\\         \dfrac{\partial f}{\partial z}(x,y,z)&=xy^2,\qquad\forall(x,y,z)\in\mathbb{R}^3. \end{split} \end{equation*}

    Ora le derivate seconde: essendo f un polinomio, è in particolare di classe \mathcal{C}^2(\mathbb{R}^2), pertanto, dal teorema di Schwarz 1.11, basterà calcolare sei derivate, anziché nove. In ogni punto (x,y,z)\in\mathbb{R}^3, si ha

        \begin{equation*}     \begin{split}         &\dfrac{\partial^2f}{\partial x^2}(x,y,z)=6x,\qquad \dfrac{\partial^2f}{\partial x\partial y}(x,y,z)=2yz=\dfrac{\partial^2f}{\partial y\partial x}(x,y,z),\\\\         &\dfrac{\partial^2f}{\partial x\partial z}(x,y,z)=y^2=\dfrac{\partial^2f}{\partial z\partial x}(x,y,z),\qquad\dfrac{\partial^2f}{\partial y^2}(x,y,z)=2xz,\\\\         &\dfrac{\partial^2f}{\partial y\partial z}(x,y,z)=2xy=\dfrac{\partial^2f}{\partial z\partial y}(x,y,z),\qquad\dfrac{\partial^2f}{\partial z^2}(x,y,z)=0.     \end{split} \end{equation*}

    Dunque, la matrice hessiana di f risulta essere

        \begin{equation*}     \nabla^2f(x,y,z)=\begin{pmatrix}     6x&2yz&y^2\\\\     2yz&2xz&2xy\\\\     y^2&2xy&0     \end{pmatrix},\qquad\forall (x,y,z)\in\mathbb{R}^3 \end{equation*}

    che è, appunto, simmetrica.

    Dato un punto critico, esso potrebbe essere un punto di massimo relativo, un punto di minimo relativo, oppure un punto di sella. Per indagare la sua natura, richiamiamo il seguente teorema.

    Teorema 1.13 (Taylor). Sia A \subseteq \mathbb{R}^n un insieme aperto e sia f \colon A \to \mathbb{R} una funzione di classe \mathcal{C}^2(A). Allora per ogni \overline{x}_0\in A esiste un intorno aperto \mathcal{U}_{\overline{x}_0} di \overline{x}_0 in cui vale

        \begin{equation*}     f(\overline{x})=f(\overline{x}_0)+\nabla f(\overline{x}_0)\cdot (\overline{x}-\overline{x}_0)+\dfrac{1}{2}(\overline{x}-\overline{x}_0)^t\cdot\nabla^2f(\overline{x}_0)\cdot(\overline{x}-\overline{x}_0)+R(\overline{x}),\qquad\forall \overline{x}\in\mathcal{U}_{\overline{x}_0}, \end{equation*}

    dove R(\overline{x})=o(||\overline{x}-\overline{x}_0||^2), ovvero soddisfa

        \begin{equation*}     \lim_{\overline{x}\to\overline{x}_0}\dfrac{R(\overline{x})}{||\overline{x}-\overline{x}_0||^2}=0. \end{equation*}

    Il polinomio

        \begin{equation*}     T^2_{f,\overline{x}_0}(\overline{x}):=f(\overline{x}_0)+\nabla f(\overline{x}_0)\cdot (\overline{x}-\overline{x}_0)+\dfrac{1}{2}(\overline{x}-\overline{x}_0)^t\cdot\nabla^2f(\overline{x}_0)\cdot(\overline{x}-\overline{x}_0) \end{equation*}

    è l’unico polinomio di secondo grado che verifica le condizioni precedenti e viene detto polinomio di Taylor di grado 2 di f con centro \overline{x}_0.

        \[\quad\]

    Dimostrazione. Vedi [1, teorema 3.15].

    Osservazione 1.14. Il teorema di Taylor afferma che il polinomio di Taylor di secondo grado di una funzione f con centro \overline{x}_0 rappresenta la parabola che meglio approssima f in un intorno di \overline{x}_0. Infatti esso è l’unico polinomio di secondo grado che, in un intorno di x_0, soddisfa

        \begin{equation*}     f(\overline{x})-T^2_{f,\overline{x}_0}=o(||\overline{x}-\overline{x}_0||^2), \end{equation*}

    cioè approssima f con un resto che è un infinitesimo di ordine superiore al secondo.

    Il prossimo corollario applica il precedente teorema al caso in cui \overline{x}_0 sia un punto critico.

    Corollario 1.15. Sia A \subseteq \mathbb{R}^n un insieme aperto, f \colon A \to \mathbb{R} una funzione di classe \mathcal{C}^2(A) e \overline{x}_0\in A un punto critico di f. Allora, esiste un intorno di \mathcal{U}_{\overline{x}_0} di \overline{x}_0 in cui vale

        \begin{equation*}     f(\overline{x})-f(\overline{x}_0)=\dfrac{1}{2}(\overline{x}-\overline{x}_0)^t\cdot\nabla^2f(\overline{x}_0)\cdot(\overline{x}-\overline{x}_0)+o(||\overline{x}-\overline{x}_0||^2),\qquad\forall \overline{x}\in\mathcal{U}_{\overline{x}_0}. \end{equation*}

        \[\quad\]

    A questo punto, mettiamo insieme i risultati e torniamo al problema iniziale dello studio dei massimi e minimi relativi di una funzione. Per definizione, sappiamo che \overline{x}_0 è un punto di massimo (minimo, rispettivamente) relativo per f se esiste un intorno \mathcal{U}_{\overline{x}_0} di \overline{x}_0 in cui vale

        \begin{equation*}     f(\overline{x})-f(\overline{x}_0)\leq 0 \qquad (f(\overline{x})-f(\overline{x}_0)\geq 0 , \ \text{risp.}),\qquad\forall \overline{x}\in\mathcal{U}_{\overline{x}_0}. \end{equation*}

    Per il teorema di Fermat 1.7, i punti di massimo o minimo relativi di una funzione differenziabile vanno ricercati tra i suoi punti critici e per il corollario 1.15 il segno di f(\overline{x})-f(\overline{x}_0) in un intorno di \overline{x}_0 dipende prevalentemente dal segno di

        \[(\overline{x}-\overline{x}_0)^t\cdot\nabla^2f(\overline{x}_0)\cdot(\overline{x}-\overline{x}_0),\]

    poiché il resto è un infinitesimo di ordine superiore. Questa osservazione sarà alla base del criterio che formuleremo per studiare la natura dei punti critici.


    Forma quadratica associata alla matrice hessiana.

    A questo punto è utile richiamare alcune definizioni dell’algebra lineare, riguardanti le forme quadratiche.

    Definizione 1.16. Sia M\in\mathcal{M}_n(\mathbb{R}) una matrice simmetrica di ordine n. Si definisce la forma quadratica associata a M la funzione Q_M:\mathbb{R}^n\rightarrow\mathbb{R} definita da

        \begin{equation*}     Q_M(v):=v^t\cdot M \cdot v,\qquad \forall v \in \mathbb{R}^n. \end{equation*}

    Una forma quadratica Q_M si dice

        \[\quad\]

    1. definita positiva se per ogni v\in\mathbb{R}^n\setminus\{\overline{0}\} vale \textcolor{red}{Q(v)>0};
    2.  

    3. definita negativa se per ogni v\in\mathbb{R}^n\setminus\{\overline{0}\} vale \textcolor{red}{Q(v)<0};
    4.  

    5. semidefinita positiva se per ogni v\in\mathbb{R}^n vale \textcolor{red}{Q(v)\geq0};
    6.  

    7. semidefinita negativa se per ogni v\in\mathbb{R}^n vale \textcolor{red}{Q(v)\leq 0};
    8.  

    9. indefinita se esistono due vettori v_+,v_-\in\mathbb{R}^n tali che \textcolor{red}{Q(v_+)>0} e \textcolor{red}{Q(v_-)<0}.

    Infine, diremo che una matrice simmetrica è definita postiva (negativa, semindefinita o indefinita, rispettivamente) se lo è la sua forma quadratica associata.

        \[\quad\]

    Mettendo insieme la definizione precedente con il corollario, otteniamo il seguente teorema.

    Teorema 1.17. Sia f:A\subseteq\mathbb{R}^n\rightarrow\mathbb{R} una funzione di classe \mathcal{C}^2(A) e sia \overline{x}_0\in A^\circ un punto critico di f. Allora valgono le seguenti implicazioni:

        \[\quad\]

    1. se \nabla^2f(\overline{x}_0) è definita positiva, \overline{x}_0 è un punto di minimo locale di f;
    2.  

    3. se \nabla^2f(\overline{x}_0) è definita negativa, \overline{x}_0 è un punto di massimo locale di f;
    4.  

    5. se \nabla^2f(\overline{x}_0) è semidefinita positiva e non nulla, \overline{x}_0 è un punto di minimo locale o un punto di sella di f;
    6.  

    7. se \nabla^2f(\overline{x}_0) è semidefinita negativa e non nulla, \overline{x}_0 è un punto di massimo locale o un punto di sella di f;
    8.  

    9. se \nabla^2f(\overline{x}_0) è indefinita, \overline{x}_0 è un punto di sella di f.

        \[\quad\]

    Dimostrazione. Dimostriamo solo il caso in cui \nabla^2f(\overline{x}_0) è definita positiva o indefinita.

    Supponiamo \nabla^2f(\overline{x}_0 definita positiva. Dal teorema di Taylor 1.13, ricordando che \overline{x}_0 è un punto critico di f, esiste un intorno \mathcal{U}_{\overline{x}_0} di \overline{x}_0 tale che

        \begin{equation*} \begin{split} f(\overline{x})-f(\overline{x}_0)&=\dfrac{1}{2}(\overline{x}-\overline{x}_0)^t\cdot\nabla^2f(\overline{x}_0)\cdot(\overline{x}-\overline{x}_0)+o(||\overline{x}-\overline{x}_0||^2)\\ &=\dfrac{1}{2}Q_{\nabla^2f(\overline{x}_0)}(\overline{x}-\overline{x}_0)+o(||\overline{x}-\overline{x}_0||^2)\qquad\forall\overline{x}\in\mathcal{U}_{\overline{x}_0}. \end{split} \end{equation*}

    Per ipotesi, \nabla^2f(\overline{x}_0) è definita positiva e o(||\overline{x}-\overline{x}_0||^2) tende a zero più velocemente di Q_{\nabla^2f(\overline{x}_0)}(\overline{x}-\overline{x}_0) per \overline{x}\to\overline{x}_0, si ha

        \begin{equation*} f(\overline{x})-f(\overline{x}_0)>0, \end{equation*}

    per ogni \overline{x} sufficientemente vicino a \overline{x}_0, ovvero \overline{x}_0 è un punto di minimo locale.

    Supponiamo ora che \nabla^2f(\overline{x}_0) sia indefinita, ovvero esistono due vettori v,w\in\mathbb{R}^n\setminus\{\overline{0}\} tali che Q_{\nabla^2f(\overline{x}_0)}(v)>0 e Q_{\nabla^2f(\overline{x}_0)}(w)<0. Applicando nuovamente il teorema di Taylor 1.13 otteniamo

        \begin{equation*} f(\overline{x}_0+tv)-f(\overline{x}_0)=\dfrac{1}{2}Q_{\nabla^2f(\overline{x}_0)}(tv)+o(||tv||^2)>0, \end{equation*}

        \begin{equation*} f(\overline{x}_0+tw)-f(\overline{x}_0)=\dfrac{1}{2}Q_{\nabla^2f(\overline{x}_0)}(tw)+o(||tw||^2)<0, \end{equation*}

    per ogni t\neq0 sufficientemente piccolo. Ciò significa che in ogni intorno di \overline{x}_0 esistono punti \overline{y},\overline{z} tali che

        \begin{equation*} f(\overline{y})<f(\overline{x}_0)<f(\overline{z}), \end{equation*}

    ovvero \overline{x}_0 è un punto di sella.

    Osservazione 1.18. Dal teorema precedente possiamo notare che solo nel caso in cui \nabla^2f(\overline{x}_0) sia definita (positivamente o negativamente) o indefinita, l’analisi della definitezza della matrice hessiana conclude lo studio del problema di determinare la natura dei punti critici.

    Al contrario, se la matrice è semidefinita ma non definita tale studio porta ad una conclusione parziale e sono necessari altri strumenti per determinare la natura del punto critico.

    Vediamo due esempi in cui lo studio della matrice hessiana non è conclusivo, poiché semidefinita, ma non definita. In un caso il punto critico in questione è un punto di sella, nell’altro un punto di minimo.

    Esempio 1.19. Sia data la seguente funzione f:\mathbb{R}^2\rightarrow\mathbb{R}, definita da

        \begin{equation*}     f(x,y):=(x^2-y-1)(1-x^2-y^2), \qquad\forall(x,y)\in\mathbb{R}^2. \end{equation*}

    Verifichiamo che i punti (1,0), (-1,0) e (0,-1) sono punti critici di f e studiamone la natura.

    Svolgimento. Osserviamo che f, essendo un polinomio è di classe almeno \mathcal{C}^2(\mathbb{R}^2). Calcoliamo il gradiente di f attraverso le sue derivate parziali:

        \[\begin{aligned}     \dfrac{\partial f}{\partial x}(x,y)&=2x(1-x^2-y^2)-2x(x^2-y-1)=2x(1-x^2-y^2-x^2+y+1)\\  &=2x(2-2x^2-y^2+y)\qquad\forall(x,y)\in\mathbb{R}^2\\\\     \dfrac{\partial f}{\partial y}(x,y)&=-(1-x^2-y^2)-2y(x^2-y-1)=3y^2-2x^2y+x^2+2y-1\qquad\forall(x,y)\in\mathbb{R}^2 \end{aligned}\]

    dunque il gradiente di f vale

        \begin{equation*}     \nabla f(x,y)=\left(2x(2-2x^2-y^2+y),3y^2-2x^2y+x^2+2y-1\right)\qquad\forall(x,y)\in\mathbb{R}^2. \end{equation*}

    È immediato osservare che i punti P_1=(1,0), P_2=(-1,0) e P_3=(0,-1) annullano il gradiente e dunque sono punti critici di f.

    Per comprendere la natura dei punti critici è utile studiare la definitezza della matrice hessiana di f valutata in essi. Calcoliamo le derivate seconde di f per costruire la matrice hessiana:

        \[\begin{aligned} \dfrac{\partial^2 f}{\partial x^2}(x,y)&=2(2-2x^2-y^2+y)-8x^2=-2(6x^2+y^2-y-2)\qquad\forall(x,y)\in\mathbb{R}^2\\ \dfrac{\partial^2 f}{\partial x\partial y}(x,y)&=-4xy+2x=\dfrac{\partial^2 f}{\partial y\partial x}(x,y)\qquad\forall(x,y)\in\mathbb{R}^2\\ \dfrac{\partial^2 f}{\partial y^2}(x,y)&=2(3y-x^2+1)\qquad\forall(x,y)\in\mathbb{R}^2. \end{aligned}\]

    Dunque la matrice hessiana di f risulta essere

        \begin{equation*}     \nabla^2f(x,y)=\begin{pmatrix}     -2(6x^2+y^2-y-2)&-4xy+2x\\\\     -4xy+2x&2(3y-x^2+1)     \end{pmatrix}\qquad\forall(x,y)\in\mathbb{R}^2. \end{equation*}

    Valutiamo la matrice in P_1, P_2 e P_3:

        \begin{equation*}     \nabla^2f(P_1)=\begin{pmatrix}     -8&2\\     2&0     \end{pmatrix},\qquad \nabla^2f(P_2)=\begin{pmatrix}     -8&-2\\     -2&0     \end{pmatrix},\qquad \nabla^2f(P_3)=\begin{pmatrix}     0&0\\     0&-4     \end{pmatrix}. \end{equation*}

    Per ciascun punto studiamo la definitezza della rispettiva matrice hessiana attraverso la forma quadratica associata.

    P_1) Sia Q_1:\mathbb{R}^2\rightarrow\mathbb{R} la forma quadratica associata a \nabla^2f(P_1). Per definizione, essa si definisce tramite

        \begin{equation*}         \begin{split}             Q_1(x,y)&=(x,y)\cdot\nabla^2f(P_1)\cdot\begin{pmatrix}         x\\         y\end{pmatrix}=(x,y)\cdot\begin{pmatrix}     -8&2\\     2&0     \end{pmatrix}\cdot\begin{pmatrix}         x\\         y\end{pmatrix}=         \\         &=(x,y)\cdot \begin{pmatrix}         -8x+2y\\         2x\end{pmatrix}=-8x^2+4xy\qquad\forall(x,y)\in\mathbb{R}^2.         \end{split}     \end{equation*}

    Si tratta di una forma quadratica indefinita, poiché possiamo prendere v_1=(1,0) e v_2=(1,3), per cui risulta

        \begin{equation*}         Q_1(v_1)=-8<0,\qquad Q_1(v_2)=4>0.     \end{equation*}

    Ciò significa che \nabla^2f(P_1) è indefinita e pertanto P_1 è un punto di sella per il teorema 1.17, visibile in figura 4.

        \[\quad\]

        \[\quad\]

    Massimi e minimi per funzioni in più variabili

    Figura 4: grafico della funzione f in un intorno del punto P_1; si vede che tale punto è di sella per f.

        \[\quad\]

        \[\quad\]

    P_2). Sia nuovamente Q_2:\mathbb{R}^2\rightarrow\mathbb{R} la forma quadratica associata a \nabla^2f(P_2):

        \begin{equation*}     \begin{split}         Q_2(x,y)&=(x,y)\cdot\nabla^2f(P_2)\cdot\begin{pmatrix}         x\\         y\end{pmatrix}= (x,y)\cdot\begin{pmatrix}     -8&-2\\     -2&0     \end{pmatrix}\cdot\begin{pmatrix}         x\\         y\end{pmatrix}\\         &=(x,y)\cdot \begin{pmatrix}         -8x-2y\\         -2x\end{pmatrix}=-8x^2-4xy\qquad\forall(x,y)\in\mathbb{R}^2.         \end{split}     \end{equation*}

    Anche Q_2 è indefinita, basti prendere questa volta v_1=(1,0) e v_2=(1,-3). Otteniamo quindi che anche P_2 è un punto di sella, ancora applicando il teorema 1.17.

    P_3). Calcoliamo anche Q_3:\mathbb{R}^2\rightarrow\mathbb{R}, la forma quadratica associata a \nabla^2f(P_3):

        \begin{equation*}     \begin{split}         Q_3(x,y)&=(x,y)\cdot\nabla^2f(P_3)\cdot\begin{pmatrix}         x\\         y\end{pmatrix}= (x,y)\cdot\begin{pmatrix}     0&0\\     0&-4     \end{pmatrix}\cdot\begin{pmatrix}         x\\         y\end{pmatrix}\\         &=(x,y)\cdot \begin{pmatrix}         0\\         -4y\end{pmatrix}=-4y^2\qquad\forall(x,y)\in\mathbb{R}^2.         \end{split}     \end{equation*}

    In questo caso Q_3 è semidefinita negativa, ma non definita negativa. Infatti Q_3(v)\leq0, per ogni v\in\mathbb{R}^2, tuttavia Q_3(1,0)=0, anche se ||(1,0)||=1\neq0. Poiché \nabla^2f(P_3) è semidefinita negativa e non nulla, lo studio della definitezza di \nabla^2f(P_3) non è conclusivo. Il teorema 1.17 afferma che P_3 potrebbe essere sia un punto di massimo relativo che un punto di sella e per determinarne la natura sarà necessario uno studio più profondo.

    Osserviamo che f(P_3)=0, dunque basterà studiare il segno di f su un qualsiasi intorno di P_3 per determinarne la natura: se il segno di f rimane costante P_3 è un punto di massimo relativo, se invece f assume valori discordi P_3 risulterà essere un punto di sella. Studiamo allora il segno di f:

    (3)   \begin{equation*}         f(x,y)\geq0\iff (x^2-y-1)(1-x^2-y^2)\geq0.     \end{equation*}

    Conviene studiare separatamente i due fattori, si veda anche la figura 5:

    (4)   \begin{equation*} x^2-y-1\geq0\iff y\leq x^2-1; \end{equation*}

    (5)   \begin{equation*}  1-x^2-y^2\geq0\iff x^2+y^2\leq 1. \end{equation*}

        \[\quad\]

    Massimi e minimi per funzioni in più variabili

    in verde, la regione del piano descritta da (4)

    Massimi e minimi per funzioni in più variabili

    in verde, la regione del piano descritta da (5)

    Figura 5: le regioni in cui i fattori presenti in (3) sono positivi.

        \[\quad\]

    Da questo concludiamo che

        \begin{equation*}         f(x,y)\geq 0\iff (x,y)\in A\cup B,     \end{equation*}

    dove

        \begin{equation*}         A=\{(x,y)\in\mathbb{R}^2\colon y\leq x^2-1\wedge x^2+y^2\leq1\}     \end{equation*}

    e

        \begin{equation*}         B=\{(x,y)\in\mathbb{R}^2\colon y\geq x^2-1\wedge x^2+y^2\geq1\}.     \end{equation*}

    La regione A \cup B è rappresentata in verde in figura 6.

    Massimi e minimi per funzioni in più variabili

    Figura 6: in verde le regioni A e B in cui f \geq 0.

    Osserviamo infine che, come rappresentato in figura 7, ogni intorno aperto di P_3 interseca sia A\cup B, dove la funzione è positiva o nulla, sia (A\cup B)^c dove la funzione è negativa o nulla. Ciò significa che in ogni intorno aperto di P_3 la funzione f cambia segno. Per quanto affermato in precedenza, anche P_3 è un punto di sella, come si evince anche dal grafico di figura 8.

    Massimi e minimi per funzioni in più variabili

    Figura 7: in turchese il luogo dei punti in cui f\geq0, mentre in rosso l’insieme in cui f\leq0. Ciascun intorno di P_3 interseca entrambe le regioni.

    Massimi e minimi per funzioni in più variabili

    Figura 8: una curva lungo cui f presenta un minimo, immagine della curva a sinistra, che in un intorno di P_3 è nella regione verde.

    Esempio 1.20. Sia data la seguente funzione f:\mathbb{R}^2\rightarrow\mathbb{R}, definita da

        \begin{equation*}     f(x,y):=x^4-xy^2+y^2,\qquad\forall (x,y)\in\mathbb{R}^2. \end{equation*}

    Calcoliamo i punti critici di f e studiamone la natura.

    Svolgimento. Osserviamo che f, essendo prodotto e somma di funzioni differenziabili è essa stessa differenziabile. Calcoliamo il gradiente di f attraverso le sue derivate parziali:

    (6)   \begin{equation*} \dfrac{\partial f}{\partial x}(x,y)=4x^3-y^2, \quad \dfrac{\partial f}{\partial y}(x,y)=-2xy+2y=2y(1-x)\qquad\forall(x,y)\in\mathbb{R}^2 \end{equation*}

    dunque il gradiente di f risulta essere

        \begin{equation*}     \nabla f(x,y)=\left(4x^3-y^2,2y(1-x)\right)\qquad\forall(x,y)\in\mathbb{R}^2. \end{equation*}

    I punti critici di f sono le coppie (x,y)\in\mathbb{R}^2, soluzione dell’equazione vettoriale \nabla f(x,y)=(0,0), ovvero le soluzioni del sistema

        \begin{equation*}     \left\{\begin{array}{l}          \dfrac{\partial f}{\partial x}(x,y)=4x^3-y^2=0\\[9pt]          \dfrac{\partial f}{\partial y}(x,y)=2y(1-x)=0.     \end{array}     \right. \end{equation*}

    Osserviamo che la seconda equazione si annulla solamente per x=1 oppure y=0. Inserendo la condizione x=1, la prima equazione diventa

        \begin{equation*}     4-y^2=0 \iff y^2=4 \iff y=\pm2. \end{equation*}

    Abbiamo dunque trovato i primi due punti critici di f: P_1=(1,2) e P_2=(1,-2), in arancio in figura 9.

    Supponendo invece y=0 nella prima equazione essa risulta x=0, trovando così l’ultimo punto critico di f, P_3=(0,0), in giallo in figura 9.

    Massimi e minimi per funzioni in più variabili

    Figura 9: i punti critici di f.

    Come al solito, calcoliamo la matrice hessiana di f e valutiamola nei punti critici appena trovati per indagare la loro natura. Non è detto, però, che questo studio sia conclusivo. Calcoliamo prima le derivate seconde di f:

        \[\begin{aligned}    & \dfrac{\partial^2 f}{\partial x^2}(x,y)=12x^2\qquad\forall(x,y)\in\mathbb{R}^2,\\[4pt]    & \dfrac{\partial^2 f}{\partial x\partial y}(x,y)=-2y =\dfrac{\partial^2 f}{\partial y\partial x}(x,y)\qquad\forall(x,y)\in\mathbb{R}^2,\\[4pt]    & \dfrac{\partial^2 f}{\partial y^2}(x,y)=-2x+2\qquad\forall(x,y)\in\mathbb{R}^2. \end{aligned}\]

    Dunque la matrice hessiana di f risulta essere

        \begin{equation*}     \nabla^2f(x,y)=\begin{pmatrix}     12x^2&-2y\\\\     -2y&-2x+2     \end{pmatrix}\qquad\forall(x,y)\in\mathbb{R}^2. \end{equation*}

    Valutiamo la matrice in P_1, P_2 e P_3:

        \begin{equation*}     \nabla^2f(1,2)=\begin{pmatrix}     12&-4\\     -4&0     \end{pmatrix},\qquad \nabla^2f(1,-2)=\begin{pmatrix}     12&4\\     4&0     \end{pmatrix},\qquad\nabla^2f(0,0)=\begin{pmatrix}     0&0\\     0&2     \end{pmatrix}. \end{equation*}

    Per ciascun punto studiamo la definitezza della rispettiva matrice hessiana attraverso la forma quadratica associata.

    P_1) Sia Q_1:\mathbb{R}^2\rightarrow\mathbb{R} la forma quadratica associata a \nabla^2f(P_1). Per definizione, essa si definisce tramite

        \begin{equation*}         \begin{split}             Q_1(x,y)&=(x,y)\cdot\nabla^2f(P_1)\cdot\begin{pmatrix}         x\\         y\end{pmatrix} \\         &=         (x,y)\cdot\begin{pmatrix}     12&-4\\     -4&0     \end{pmatrix}\cdot\begin{pmatrix}         x\\         y\end{pmatrix}\\         &=(x,y)\cdot \begin{pmatrix}         12x-4y\\         -4x\end{pmatrix}         \\         &=         12x^2-8xy\qquad\forall(x,y)\in\mathbb{R}^2.         \end{split}     \end{equation*}

    Si tratta di una forma quadratica indefinita, infatti, detti v_1=(1,0) e v_2=(1,2), risulta

        \begin{equation*}         Q_1(v_1)=12>0,\qquad Q_1(v_2)=-4<0.     \end{equation*}

    Ciò significa che \nabla^2f(P_1) è indefinita e pertanto dal teorema 1.17, P_1 è un punto di sella, visibile anche dal grafico in figura 10.

        \[\quad\]

        \[\quad\]

    Massimi e minimi per funzioni in più variabili

    Figura 10: rappresentazione di f in un intorno del punto critico P_1.

        \[\quad\]

        \[\quad\]

    P_2) Sia nuovamente Q_2:\mathbb{R}^2\rightarrow\mathbb{R} la forma quadratica associata a \nabla^2f(P_2):

        \begin{equation*}     \begin{split}         Q_2(x,y)&=(x,y)\cdot\nabla^2f(P_2)\cdot\begin{pmatrix}         x\\         y\end{pmatrix}= (x,y)\cdot\begin{pmatrix}     12&4\\     4&0     \end{pmatrix}\cdot\begin{pmatrix}         x\\         y\end{pmatrix}\\         &=(x,y)\cdot \begin{pmatrix}         12x4y\\         4x\end{pmatrix}=12x^2+8xy\qquad\forall(x,y)\in\mathbb{R}^2.         \end{split}     \end{equation*}

    È possibile dimostrare l’indefinitezza di Q_2, prendendo come vettori test v_1=(1,0) e v_2=(1,-2). Applicando nuovamente il teorema 1.17, troviamo che P_2 è un punto di sella.

    P_3) Infine, detta Q_3:\mathbb{R}^2\rightarrow\mathbb{R} la forma quadratica associata a \nabla^2f(P_3), si ha

        \begin{equation*}         \begin{split}             Q_3(x,y)&=(x,y)\cdot\nabla^2f(P_3)\cdot\begin{pmatrix}         x\\         y\end{pmatrix}=(x,y)\cdot\begin{pmatrix}     0&0\\     0&2     \end{pmatrix}\cdot\begin{pmatrix}         x\\         y\end{pmatrix}=(x,y)\cdot \begin{pmatrix}         0\\         2y\end{pmatrix}=2y^2\qquad\forall(x,y)\in\mathbb{R}^2.         \end{split}     \end{equation*}

    Abbiamo già visto essere una forma quadratica non nulla, semidefinita positiva, ma non definita positiva. Il teorema ?? implica che P_1 può sia essere un punto di sella, sia un punto di minimo relativo e lo studio della matrice hessiana non è in grado di fornirci informazioni più dettagliate.

    Osserviamo che anche in questo caso f(P_3)=0, dunque la natura di P_3 può essere determinata studiando il segno di f su un intorno sufficientemente piccolo di P_3. Dato \mathcal{U}_{P_3}\coloneqq\{(x,y)\in\mathbb{R}^2\mid x<1\}, l’intorno di P_3 rappresentato in azzurro in figura 11, si ha

        \begin{equation*}         f(x,y)=x^4+y^2(1-x)\geq x^4\geq0=f(P_3), \qquad\forall(x,y)\in\mathcal{U}_{P_3},     \end{equation*}

    poiché (x,y)\in\mathcal{U}_{P_3} implica 1-x<0. Abbiamo dunque trovato un intorno aperto \mathcal{U}_{P_3} di P_3, in cui f(x,y)\geq f(P_3), \forall (x,y)\in\mathcal{U}_{P_3}, verificando così, tramite definizione, che P_3 è un punto di minimo relativo per f, come si evince anche dal grafico di f riportato in figura 12.

        \[\quad\]

    Massimi e minimi per funzioni in più variabili

        \[\quad\]

        \[\quad\]

    Osserviamo che vale anche il viceversa del teorema 1.17, ma in forma più debole.

    Proposizione 1.21. Sia f:A\subseteq\mathbb{R}^n\rightarrow\mathbb{R} una funzione di classe \mathcal{C}^2(A) e sia \overline{x}_0\in A^\circ un punto critico di f. Allora valgono le seguenti considerazioni:

        \[\quad\]

    1. se \overline{x}_0 è un punto di minimo locale di f, allora \nabla^2f(\overline{x}_0) è semidefinita positiva;
    2.  

    3. se \overline{x}_0 è un punto di massimo locale di f, allora \nabla^2f(\overline{x}_0) è semidefinita negativa;
    4.  

    5. se \overline{x}_0 è un punto di sella di f, \nabla^2f(\overline{x}_0) è indefinita o semidefinita.

        \[\quad\]

    Dimostrazione. Segue dal Teorema di Fermat 1.7 e di Taylor 1.13.

    Il teorema 1.17 mostra come nello studio dei punti di massimo e minimo di una funzione sia essenziale indagare la definitezza della matrice hessiana valutata nei punti critici della funzione. I prossimi risultati forniscono degli strumenti in tal senso.

    Definizione 1.22. Sia A\in\mathcal{M}_n(\mathbb{R}) una matrice. Un autovalore di A è uno scalare \lambda\in\mathbb{R} per cui esiste un vettore v\in\mathbb{R}^n\setminus\{\overline{0}\} tale che

        \[A v=\lambda v.\]

    Tale vettore v viene definito autovettore associato all’autovalore \lambda.

    Proposizione 1.23. Sia A\in\mathcal{M}_n(\mathbb{R}) una matrice. Allora i suoi autovalori sono radici del polinomio caratteristico, ovvero il polinomio di grado n

        \begin{equation*}     p_A(\lambda):=\det(A-\lambda \operatorname{Id}). \end{equation*}

        \[\quad\]

    Dimostrazione. Dalla definizione di autovalore si ha che esiste un vettore v\in\mathbb{R}^n\setminus\{\overline{0}\} tale che

        \[0=Av-\lambda v=(A-\lambda \operatorname{Id})v.\]

    Essendo v non nullo, la matrice A-\lambda\operatorname{Id} è singolare o, equivalentemente \det(A-\lambda \operatorname{Id})=0.

    Osservazione 1.24. Se A\in\mathcal{M}_n(\mathbb{R}) è la matrice

        \begin{equation*}     A=\begin{pmatrix}     a_{11}&a_{12}&\dots&a_{1n}\\     a_{21}&a_{22}&\dots&a_{2n}\\     \vdots&\vdots&\ddots&\vdots\\     a_{n1}&a_{n2}&\dots&a_{nn}\\     \end{pmatrix}, \end{equation*}

    il suo polinomio caratteristico è

        \begin{equation*}     p_A(\lambda):=\det\begin{pmatrix}     a_{11}-\lambda&a_{12}&\dots&a_{1n}\\\\     a_{21}&a_{22}-\lambda&\dots&a_{2n}\\\\     \vdots&\vdots&\ddots&\vdots\\     a_{n1}&a_{n2}&\dots&a_{nn}-\lambda\\     \end{pmatrix}. \end{equation*}

    La prossima proposizione afferma che è possibile esprimere la definitezza di una matrice mediante i suoi autovalori. Ricordiamo che, grazie al teorema spettrale [1, §6.4], una matrice simmetrica è sempre diagonalizzabile (ovvero simile ad una matrice diagonale in cui gli elementi sulla diagonale sono proprio gli autovalori della matrice).

    Proposizione 1.25. Sia M\in\mathcal{M}_n(\mathbb{R}) una matrice simmetrica. Allora:

        \[\quad\]

    1. M è minimodefinita positiva se e solo se tutti i suoi autovalori sono positivi;
    2.  

    3. M è definita negativa se e solo se tutti i suoi autovalori sono negativi;
    4.  

    5. M è semidefinita positiva se e solo se tutti i suoi autovalori sono positivi o nulli;
    6.  

    7. M è semidefinita negativa se e solo se tutti i suoi autovalori sono negativi o nulli;
    8.  

    9. M è indefinita se e solo se possiede autovalori di segno discorde.

        \[\quad\]

    Dimostrazione. Vedi [1, teorema 3.20].

    Solitamente, per calcolare esplicitamente gli autovalori di una matrice, occorre trovare le radici del suo polinomio caratteristico, scrivendo così la matrice in forma diagonale. Si ottiene cioè una matrice simile a quella di partenza, in cui sulla diagonale compaiono gli autovalori trovati. La diagonalizzazione è un processo talvolta lungo e tedioso, il prossimo criterio permetterà di ottenenere un risultato sufficiente per i nostri scopi, senza però calcolare esplicitamente gli autovalori della matrice.

    Definizione 1.26. Sia M\in\mathcal{M}_n(\mathbb{R}) una matrice. Definiamo:

        \[\quad\]

    1. i minori nord-ovest di ordine k, come i determinanti delle sottomatrici ottenute eliminando da A le ultime n-k righe e le ultime n-k colonne;
    2.  

    3. i minori principali di ordine k, come i determinanti delle sottomatrici di ordine k ottenute eliminando n-k righe e colonne dello stesso indice (non necessariamente le ultime).

        \[\quad\]

    Esempio 1.27. Consideriamo la matrice simmetrica A\in\mathcal{M}_3(\mathbb{R}),

        \begin{equation*}     A=\begin{pmatrix}     0&0&1\\     0&-1&-2\\     1&-2&-1     \end{pmatrix}. \end{equation*}

    Mostriamo prima i suoi minori nord-ovest, ovvero i determinanti delle sottomatrici di ordine k in cui vengono eliminate le ultime 3-k righe e colonne. Vi saranno in tutto tre sottomatrici, a cui corrispondono altrettanti minori nord-ovest:

        \[\quad\]

    1. la sottomatrice A_1 di ordine 1, ottenuta eliminando da A le ultime 2 righe e colonne: Massimi e minimi per funzioni in più variabili
    2.  

    3. la sottomatrice A_2 di ordine 2, ottenuta eliminando da A l’ultima riga e colonna: Massimi e minimi per funzioni in più variabili
    4.  

    5. la matrice A stessa Massimi e minimi per funzioni in più variabili

    Per quanto riguarda i minori principali, invece, ne troviamo sette, corrispondenti ad altrettante sottomatrici: tre di ordine 1, tre di ordine 2 e uno di ordine 3.

        \[\quad\]

    1. Quelli di ordine 1 sono Massimi e minimi per funzioni in più variabili
    2.  

    3. Quelli di ordine 2 sono Massimi e minimi per funzioni in più variabili Massimi e minimi per funzioni in più variabili
    4.  

    5. Infine, l’unico minore principale di ordine 3 è la matrice A stessa Massimi e minimi per funzioni in più variabili

    Teorema 1.28 (Criterio di Sylvester). Sia M\in\mathcal{M}_n(\mathbb{R}) una matrice simmetrica. Allora

        \[\quad\]

    1. M è definita positiva se e solo se tutti i suoi minori nord-ovest sono positivi;
    2.  

    3. M è definita negativa se e solo se tutti i suoi minori nord-ovest di ordine pari sono positivi, mentre quelli di ordine dispari sono negativi;
    4.  

    5. M è semidefinita positiva se e solo se tutti i suoi minori principali sono positivi o nulli;
    6.  

    7. M è semidefinita negativa se e solo se tutti i suoi minori principali di ordine pari sono positivi o nulli e quelli di ordine dispari sono negativi o nulli;
    8.  

    9. M è indefinita se non si verifica nessuno dei casi precedenti.

        \[\quad\]

    Si veda [1, teorema 3.19] per la dimostrazione dei punti 1, 2 e 5, e questo link per i punti 3 e 4.

    Esempio 1.29. Studiamo la definitezza della matrice A dell’esempio precedente, alla luce del criterio appena formulato.

    Analizziamo d’apprima i minori nord-ovest di A. Essi sono rispettivamente

        \begin{equation*}     \det(A_1)=0,\qquad\det(A_2)=0,\qquad\det(A_3)=1. \end{equation*}

    Poiché i minori non sono tutti strettamente positivi, né a segni alterni, deduciamo dal teorema 1.28 che la matrice A non è definita positiva, né definita negativa. Inoltre, siccome \det(A_3)=1>0, A non può essere nemmeno semidefinita negativa, poiché in tal caso si avrebbe \det(A_3)\leq0.

    Rimangono solo due possibilità: A semidefinita positiva, oppure A indefinita. Indaghiamo i minori principali:

    (7)   \begin{gather*} \det(0)=0,\qquad\det(-1)=-1,\qquad\det(-1)=-1; \\[4pt] \det\begin{pmatrix} 0&0\\ 0&1 \end{pmatrix}=0,\qquad\det\begin{pmatrix} 0&1\\ 1&-1 \end{pmatrix}=-1,\qquad\det\begin{pmatrix} -1&-2\\ -2&-1 \end{pmatrix}=-3; \\[4pt] \det(A)=1. \end{gather*}

    Se fosse il primo caso, tutti i minori principali dovrebbero risultare positivi o nulli, tuttavia questo non accade. Infatti, abbiamo trovato quattro minori principali negativi. Pertanto, A è indefinita.

    Osservazione 1.30. Ovviamente, non vi è nulla di particolare nei minori nord-ovest nel criterio di Sylvester ed un criterio analogo può essere formulato mediante i minori sud-est, ovvero i determinanti delle sottomatrici ottenute eliminando le prime n-k righe e le prime n-k colonne. Questo segue dalla simmetria della matrice di partenza. Mostriamo, nel caso di \mathbb{R}^2, come le due richieste siano analoghe.

    Data una generica matrice simmetrica A\in\mathcal{M}_2(\mathbb{R}),

        \begin{equation*}     A=\begin{pmatrix}     a&b\\     b&c     \end{pmatrix}, \end{equation*}

    i suoi minori nord-ovest sono a e \det(A), mentre i minori sud-est corrispondono a c e nuovamente \det(A). Richiedere che i minori nord-ovest siano positivi equivale al seguente sistema

        \begin{equation*}     \left\{\begin{array}{l}          a>0\\\\     ac-b^2>0,     \end{array}\right.\iff c>\dfrac{b^2}{a}>0. \end{equation*}

    Si vede che, scrivendo la condizione per i minori di sud-est, si ottiene la stessa conclusione. L’equivalenza delle due può essere attribuita al fatto che sono la stessa richiesta, modulo un cambiamento di base. Da questo si intuisce che l’equivalenza sia valida in ogni dimensione.

    Riformuliamo ora il teorema 1.17 alla luce del teorema 1.28.

    Corollario 1.31. Sia f:A\subseteq\mathbb{R}^n\rightarrow\mathbb{R} una funzione di classe \mathcal{C}^2(A), sia \overline{x}_0\in A^\circ un punto critico di f e \nabla^2f(\overline{x}_0) la matrice hessiana di f in \overline{x}_0. Vale allora:

        \[\quad\]

    1. \overline{x}_0 è un punto di minimo locale per f se \nabla^2f(\overline{x}_0) è definita positiva, cioè se tutti i suoi minori nord-ovest sono positivi;
    2.  

    3. \overline{x}_0 è un punto di massimo locale per f se \nabla^2f(\overline{x}_0) è definita negativa, cioè se tutti i suoi minori nord-ovest di ordine pari sono positivi e i minori nord-ovest di ordine dispari sono negativi;
    4.  

    5. \overline{x}_0 è un punto di sella per f se \nabla^2f(\overline{x}_0) è indefinita.

        \[\quad\]

    In quanto segue, caratterizziamo le situazioni più frequenti, esplicitando lo studio di matrici hessiane di funzioni con dominio in \mathbb{R}^2 e in \mathbb{R}^3.

    Proposizione 1.32 (Criterio di Sylvester in \mathbb{R}^2). Sia \nabla^2f(x_0,y_0)\in\mathcal{M}_2(\mathbb{R}) la matrice hessiana di una funzione f\in\mathcal{C}^2(\mathbb{R}^2), con (x_0,y_0)\in\mathbb{R}^2 punto critico di f:

        \begin{equation*}     \nabla^2f(x_0,y_0)=\begin{pmatrix}     f_{xx}(x_0,y_0)&f_{xy}(x_0,y_0)\\\\     f_{xy}(x_0,y_0)&f_{yy}(x_0,y_0)     \end{pmatrix}. \end{equation*}

    Allora \nabla^2f(x_0,y_0) è:

        \[\quad\]

    1. definita positiva se e solo se

          \begin{equation*}         \left\{\begin{array}{l}              f_{xx}(x_0,y_0)>0\\\\               \det(\nabla^2f(x_0,y_0))>0.         \end{array}         \right.     \end{equation*}

      In questo caso (x_0,y_0) è un punto di minimo;

    2.  

    3. definita negativa se e solo se

          \begin{equation*}         \left\{\begin{array}{l}              f_{xx}(x_0,y_0)<0\\\\              \det(\nabla^2f(x_0,y_0))>0.         \end{array}         \right.     \end{equation*}

      In questo caso (x_0,y_0) è un punto di massimo;

    4.  

    5. semidefinita positiva se e solo se

          \begin{equation*}         \left\{\begin{array}{l}              f_{xx}(x_0,y_0)\geq0\\\\              f_{yy}(x_0,y_0)\geq0\\\\              \det(\nabla^2f(x_0,y_0))\geq0;         \end{array}         \right.     \end{equation*}

    6.  

    7. semidefinita negativa se e solo se

          \begin{equation*}         \left\{\begin{array}{l}              f_{xx}(x_0,y_0)\leq0\\\\              f_{yy}(x_0,y_0)\leq0\\\\              \det(\nabla^2f(x_0,y_0))\geq0;         \end{array}         \right.     \end{equation*}

    8.  

    9. indefinita se e solo se

          \begin{equation*}         \det(\nabla^2f(x_0,y_0))<0.     \end{equation*}

      In questo caso (x_0,y_0) è un punto di sella.

    Proposizione 1.33 (Criterio di Sylvester in \mathbb{R}^3). Sia \nabla^2f(x_0,y_0,z_0)\in\mathcal{M}_3(\mathbb{R}) la matrice hessiana di una funzione f\in\mathcal{C}^2(\mathbb{R}^3), con (x_0,y_0,z_0)\in\mathbb{R}^3 punto critico di f:

        \begin{equation*}     \nabla^2f(x_0,y_0,z_0)=\begin{pmatrix}     f_{xx}(x_0,y_0,z_0)&f_{xy}(x_0,y_0,z_0)&f_{xz}(x_0,y_0,z_0)\\\\     f_{xy}(x_0,y_0,z_0)&f_{yy}(x_0,y_0,z_0)&f_{yz}(x_0,y_0,z_0)\\\\     f_{xz}(x_0,y_0,z_0)&f_{yz}(x_0,y_0,z_0)&f_{zz}(x_0,y_0,z_0)     \end{pmatrix}. \end{equation*}

    Allora \nabla^2f(x_0,y_0,z_0) è2

        \[\quad\]

    1. definita positiva se e solo se

          \begin{equation*}         \left\{\begin{array}{l}              f_{xx}>0\\\\              f_{xx}f_{yy}-f_{xy}^2>0\\\\              \det(\nabla^2f(x_0,y_0,z_0))>0.         \end{array}         \right.     \end{equation*}

      In questo caso (x_0,y_0,z_0) è un punto di minimo;

    2.  

    3. definita negativa se e solo se

          \begin{equation*}         \left\{\begin{array}{l}              f_{xx}<0\\\\              f_{xx}f_{yy}-f_{xy}^2>0\\\\              \det(\nabla^2f(x_0,y_0,z_0))<0.         \end{array}         \right.     \end{equation*}

      In questo caso (x_0,y_0,z_0) è un punto di massimo;

    4.  

    5. semidefinita positiva se e solo se

          \begin{equation*}         \left\{\begin{array}{l}              f_{xx},f_{yy},f_{zz}\geq0\\\\              f_{xx}f_{yy}-f_{xy}^2,f_{xx}f_{zz}-f_{xz}^2,f_{yy}f_{zz}-f_{yz}^2\geq0\\\\              \det(\nabla^2f(x_0,y_0,z_0))\geq0;         \end{array}         \right.     \end{equation*}

    6.  

    7. semidefinita negativa se e solo se

          \begin{equation*}         \left\{\begin{array}{l}              f_{xx},f_{yy},f_{zz}\leq0\\\\              f_{xx}f_{yy}-f_{xy}^2,f_{xx}f_{zz}-f_{xz}^2,f_{yy}f_{zz}-f_{yz}^2\geq0\\\\              \det(\nabla^2f(x_0,y_0,z_0))\leq0;         \end{array}         \right.     \end{equation*}

    8.  

    9. indefinita se e solo se non si verificano i casi precedenti. In questo caso (x_0,y_0,z_0) è un punto di sella.

        \[\quad\]

    Quando la matrice hessiana è semidefinita ma non definita (in questa situazione è condizione necessaria che la matrice hessiana sia singolare) la teoria fin qui sviluppata non è efficace e non vi è un metodo universale che possa applicarsi ad ogni situazione. Pertanto, si deve procedere con considerazioni specifiche. In precedenza, abbiamo mostrato come in tali casi si possa verificare esplicitamente che il punto critico soddisfa la definizione di punto di massimo o minimo o di sella. Nell’ultimo caso è utile sottolineare come vi sia un altro modo (equivalente alla definizione) per ottenere che la natura di tale punto critico è proprio un punto di sella. Esso consiste nel dimostrare che, considerando la restrizione della funzione in questione lungo due cammini distinti passanti per il punto critico, esso risulta punto di massimo da una parte e punto di minimo dall’altra, intesi per la funzione di una variabile ottenuta come restrizione. In questo modo, il punto critico è necessariamente un punto di sella. Il prossimo esempio mostra concretamente il procedimento da seguire.

    Esempio 1.34. Sia data la seguente funzione f:\mathbb{R}^2\rightarrow\mathbb{R}, definita da

        \begin{equation*}     f(x,y):=x^4+y^4-2(x-y)^2\qquad\forall(x,y)\in\mathbb{R}^2. \end{equation*}

    Calcoliamo i punti critici di f e studiamone la natura.

    Svolgimento. f è un polinomio, dunque è possibile derivare inifinite volte f con continuità. Iniziamo con le derivate parziali per trovare gradiente e punti critici di f:

        \[\begin{aligned}     &\dfrac{\partial f}{\partial x}(x,y)=4x^3-4(x-y)\qquad\forall(x,y)\in\mathbb{R}^2\\\\     &\dfrac{\partial f}{\partial x}(x,y)=4y^3+4(x-y)\qquad\forall(x,y)\in\mathbb{R}^2, \end{aligned}\]

    da cui

        \begin{equation*}     \nabla f(x,y)=4(x^3-(x-y),y^3+(x-y))\qquad\forall(x,y)\in\mathbb{R}^2. \end{equation*}

    I punti critici di f, ovvero i punti (x,y)\in\mathbb{R}^2 che annullano il gradiente, sono le soluzioni del seguente sistema

        \begin{equation*}     \left\{\begin{array}{l}           \dfrac{\partial f}{\partial x}(x,y)=0\\\\           \dfrac{\partial f}{\partial x}(x,y)=0     \end{array}     \right.\Leftrightarrow     \left\{\begin{array}{l}           x^3-(x-y)=0\\\\           y^3+(x-y)=0     \end{array}     \right.\Leftrightarrow     \left\{\begin{array}{l}           x^3=x-y\\\\           -y^3=x-y     \end{array}     \right.\Leftrightarrow     \left\{\begin{array}{l}           x^3=-y^3\\\\           y^3=y-x.     \end{array}     \right. \end{equation*}

    La prima equazione induce il vincolo x=-y, che inserito nella seconda produce y^3=2y, che ha come soluzioni

        \begin{equation*}   y=0\lor y=\pm\sqrt{2}. \end{equation*}

    Dalla relazione x=-y, troviamo i tre punti critici di f, rappresentati in rosso in figura 13.

        \begin{equation*}     P_1=(0,0)\qquad P_2=(\sqrt{2},-\sqrt{2}),\qquad P_3=(-\sqrt{2},\sqrt{2}). \end{equation*}

        \[\quad\]

        \[\quad\]

    Figura 13: la retta x=-y e, in rosso, i tre punti critici di f.

        \[\quad\]

        \[\quad\]

    Calcoliamo ora le derivate seconde di f per studiare la natura di questi critici attraverso la definitezza della matrice hessiana:

        \[\begin{aligned}     &\dfrac{\partial^2 f}{\partial x^2}(x,y)=12x^2-4\qquad\forall(x,y)\in\mathbb{R}^2,\\[4pt]     &\dfrac{\partial^2 f}{\partial x\partial y}(x,y)=4=\dfrac{\partial^2 f}{\partial y\partial x}(x,y)\qquad\forall(x,y)\in\mathbb{R}^2,\\[4pt]     &\dfrac{\partial^2 f}{\partial y^2}(x,y)=12y^2-4\qquad\forall(x,y)\in\mathbb{R}^2, \end{aligned}\]

    da cui

        \begin{equation*}     \nabla^2f(x,y)=\begin{pmatrix} 12x^2-4&4\\\\ 4&12x^2-4     \end{pmatrix}\qquad\forall(x,y)\in\mathbb{R}^2. \end{equation*}

    Valutiamo ora la matrice hessiana su ciascuno dei punti critici:

        \begin{equation*}     \nabla^2f(P_1)=\begin{pmatrix} -4&4\\\\ 4&-4     \end{pmatrix},\qquad\nabla^2f(P_2)=\nabla^2f(P_3)=\begin{pmatrix} 20&4\\\\ 4&20     \end{pmatrix} \end{equation*}

    Osserviamo immediatamente che \nabla^2f(P_i) è definita positiva per i=2,3, infatti vale

    (8)   \begin{equation*} f_{xx}(P_i)>0, \qquad \det(\nabla^2f(P_i))>0 \end{equation*}

    e la conclusione segue dal criterio di Sylvester in \mathbb{R}^2 (proposizione 1.32). Ciò significa che P_2 e P_3 sono entrambi punti di minimo relativo per f, come si evince dal grafico di f rappresentato in figura 14.

        \[\quad\]

        \[\quad\]

        \[\quad\]

    Concentriamoci ora su P_1. Notando che \det(\nabla^2f(P_1))=0, ovvero la matrice hessiana relativa a P_1 è singolare ed essendo in \mathbb{R}^2, concludiamo che essa non è definita, ma solo semidefinita3 e non possiamo determinare la natura di P_1 mediante lo studio di \nabla^2f(P_1).

    Proviamo allora a dimostrare che P_1 è un punto di sella, esibendo due cammini distinti \Gamma_1 e \Gamma_2 passanti per P_1, per cui P_1 risulterà rispettivamente punto di massimo e punto di minimo per le restrizioni di f, f|_{\Gamma_1} e f|_{\Gamma_2}, come si vede anche in figura 15. Più precisamente, se \gamma_1 e \gamma_2 sono le parametrizzazioni di \Gamma_1 e \Gamma_2, con P_1=\gamma_1(0)=\gamma_2(0), dobbiamo mostrare che 0 sia punto di massimo e punto di minimo di f\circ\gamma_i, i=1,2. Siano \Gamma_1 e \Gamma_2 le rette date dalle equazioni

        \[\begin{aligned}     \Gamma_1\coloneqq&\{(x,y)\in\mathbb{R}^2 \mid y=0\},\\     \Gamma_2\coloneqq&\{(x,y)\in\mathbb{R}^2 \mid x=y\}, \end{aligned}\]

    rispettivamente di parametrizzazioni

        \begin{equation*}     \gamma_1:\mathbb{R}\ni t\mapsto (t,0)\in\mathbb{R}^2,\qquad \gamma_2:\mathbb{R}\ni t\mapsto (t,t)\in\mathbb{R}^2, \end{equation*}

    rappresentate in rosso e in nero in figura 15. Notiamo che P_1=\gamma_1(0)=\gamma_2(0). Si ha dunque

        \begin{equation*}     (f\circ\gamma_1)(t)=f(t,0)=t^4-2t^2,\qquad\forall t\in\mathbb{R}, \end{equation*}

    da cui

        \begin{equation*} f'(0)=0,\quad f''(0)=-4<0, \end{equation*}

    dunque 0 è un punto di massimo per f\circ\gamma_1, mentre

        \begin{equation*}     (f\circ\gamma_2)(t)=f(t,t)=2t^4\qquad\forall t\in\mathbb{R} \end{equation*}

    che ha minimo in 0.

    Dunque abbiamo provato che P_1 rappresenta sia un punto di minimo, sia un punto di massimo per f se ristretta su opportuni cammini. Questo comportamento è possibile solo se P_1 è un punto di sella.    


    1. per alleggerire la notazione scriveremo f_{ij} per indicare f_{ij}(x_0,y_0,z_0)
    2.  

      1. In \mathbb{R}^n, con n\geq3, la singolarità della matrice hessiana non equivale alla semidefinitezza della matrice. Infatti, la matrice potrebbe avere comunque un autovalore nullo e due atovalori di segno opposto e in tal caso la matrice risulta indefinita e il punto critico di sella. In dimensione 2, questo non è possibile

     

     

    Scarica la teoria

    Ottieni il documento contenente pillole teoriche sui massimi e minimi di funzioni in più variabili.

     
     

    Esercizio guida

        \[\quad\]

    Esercizio 2.1 (esercizio guida). Sia f:A\subseteq\mathbb{R}^n\rightarrow\mathbb{R} una funzione di classe \mathcal{C}^2(A). Studiare la natura dei punti critici di f su A.

    Svolgimento.

    1. La prima cosa da fare è trovare i punti critici di f, ovvero i punti (x_1,...,x_n)\in A che verificano

          \[\nabla f(x_1,...,x_n)=(0,...,0).\]

      Essi saranno le soluzioni del sistema

          \begin{equation*}     \left\{\begin{array}{l}          \dfrac{\partial f}{\partial x_1}(x_1,...,x_n)=0  \\\\           \dfrac{\partial f}{\partial x_2}(x_1,...,x_n)=0\\           ...\\           ...\\           ...\\\\           \dfrac{\partial f}{\partial x_n}(x_1,...,x_n)=0.     \end{array}\right. \end{equation*}

      Siano P_1,...,P_k\in A i punti critici di f su A trovati risolvendo il precedente sistema.

    2.  

    3. Calcoliamo tutte le derivate seconde di f. Grazie al teorema di Schwarz (teorema 1.11), \dfrac{\partial^2f}{\partial x_i\partial x_j}=\dfrac{\partial^2f}{\partial x_j\partial x_i} per i,j=1,...,n , dunque basterà calcolare \dfrac{n\cdot(n+1)}{2} derivate seconde, anziché n^2. Questo permetterà di calcolare la matrice hessiana di f in un generico punto \overline{x}=(x_1,...,x_n)

          \begin{equation*}     \nabla^2f(\overline{x})    \coloneqq    \begin{pmatrix}     \dfrac{\partial^2f}{\partial x_1^2}(\overline{x})&\dfrac{\partial^2f}{\partial x_1\partial x_2}(\overline{x})&\cdot\cdot\cdot&\dfrac{\partial^2f}{\partial x_1\partial x_n}(\overline{x})\\\\     \dfrac{\partial^2f}{\partial x_2\partial x_1}(\overline{x})&\dfrac{\partial^2f}{\partial x_2^2}(\overline{x})&\cdots&\dfrac{\partial^2f}{\partial x_2\partial x_n}(\overline{x})\\\\     \vdots&\vdots&\ddots&\vdots\\\\     \dfrac{\partial^2f}{\partial x_n\partial x_1}(\overline{x})&\dfrac{\partial^2f}{\partial x_n\partial x_2}(\overline{x})&\cdot\cdot\cdot&\dfrac{\partial^2f}{\partial x_n^2}(\overline{x})     \end{pmatrix}. \end{equation*}

    4.  

    5. Valutare la matrice hessiana nei punti critici P_1,...,P_k

          \begin{equation*}     \nabla^2f(P_j)     \coloneqq     \begin{pmatrix}     \dfrac{\partial^2f}{\partial x_1^2}(P_j)&\cdot\cdot\cdot&\dfrac{\partial^2f}{\partial x_1\partial x_n}(P_j)\\\\     \dfrac{\partial^2f}{\partial x_2\partial x_1}(P_j)&\cdots&\dfrac{\partial^2f}{\partial x_2\partial x_n}(P_j)\\\\     \vdots&\ddots&\vdots\\\\     \dfrac{\partial^2f}{\partial x_n\partial x_1}(P_j)&\cdot\cdot\cdot&\dfrac{\partial^2f}{\partial x_n^2}(P_j)     \end{pmatrix}     \quad \quad     \forall j=1,...,k. \end{equation*}

    6.  

    7. Per ciascuno dei punti critici P_1,...,P_k, studiare la definitezza della matrice hessiana valutata in tale punto, per applicare il teorema 1.17 e stabilire la natura dei punti critici. È possibile sia calcolare esplicitamente i suoi autovalori, sia procedere con il criterio di Sylvester 1.28.
    8.  

    9. Nel caso una matrice hessiana fosse semidefinita, ma non definita (positivamente o negativamente) non è possibile dedurre la natura di tale punto critico utilizzando il teorema 1.17, pertanto bisogna analizzare il problema diversamente.

     
     

    Esercizi

        \[\quad\]

    Esercizio 3.1  (\bigstar\bigstar\largewhitestar\largewhitestar\largewhitestar). Sia data la seguente funzione f:\mathbb{R}^2\rightarrow\mathbb{R}, definita da

        \begin{equation*}     f(x,y)=\sin(x)+\sin(y)\qquad\forall(x,y)\in\mathbb{R}^2. \end{equation*}

    Calcolare i punti critici di f e studiarne la natura.

    Svolgimento 1.

    Proponiamo inizialmente uno svolgimento che permette di risolvere l’esercizio in maniera immediata. Poiché -1\leq\sin(t)\leq 1, per ogni t\in\mathbb{R}, risulta

        \begin{equation*} -2\leq f(x,y)\leq 2\qquad\forall (x,y)\in\mathbb{R}^2. \end{equation*}

    Osserviamo che f è di classe \mathcal{C}^1(\mathbb{R}^2), dunque esistono e sono continue le sue derivate parziali. Troviamo i punti critici di f in \mathbb{R}^2.

        \begin{equation*}    (0,0)=\nabla f(x,y)=(\cos(x),\sin(y))\iff (x,y)=\left(\dfrac{\pi}{2}+j\pi,\dfrac{\pi}{2}+k\pi\right),\qquad\forall j,k\in\mathbb{Z}. \end{equation*}

    Supponiamo j,k\in2\mathbb{Z}, allora

        \begin{equation*} f\left(\dfrac{\pi}{2}+j\pi,\dfrac{\pi}{2}+k\pi\right)=f\left(\dfrac{\pi}{2},\dfrac{\pi}{2}\right)=1+1=2, \end{equation*}

    dunque \left(\dfrac{\pi}{2}+j\pi,\dfrac{\pi}{2}+k\pi\right) sono punti di massimo assoluto per f se j,k\in2\mathbb{Z}. Supponiamo invece j,k\in2\mathbb{Z}+1, allora

        \begin{equation*} f\left(\dfrac{\pi}{2}+j\pi,\dfrac{\pi}{2}+k\pi\right)=f\left(\dfrac{3\pi}{2},\dfrac{3\pi}{2}\right)=-1-1=-2, \end{equation*}

    dunque \left(\dfrac{\pi}{2}+j\pi,\dfrac{\pi}{2}+k\pi\right) sono punti di minimo assoluto per f se j,k\in2\mathbb{Z}+1. Infine, se j\in2\mathbb{Z} e k\in2\mathbb{Z}+1 (o viceversa) \sin(x) assume il proprio massimo in \dfrac{\pi}{2}+j\pi, mentre \sin(y) il proprio minimo in \dfrac{\pi}{2}+k\pi (o viceversa, rispettivamente). Ciò implica necessariamente che i punti (x,y)=\left(\dfrac{\pi}{2}+j\pi,\dfrac{\pi}{2}+k\pi\right) sono di sella se j\in2\mathbb{Z} e k\in2\mathbb{Z}+1 o se j\in2\mathbb{Z}+1 e k\in2\mathbb{Z}.


    Svolgimento 2.

    Per scopi didattici, mostriamo come giungere alla soluzione anche utilizzando lo studio della segnatura della matrice hessiana nei punti critici. Osserviamo che f è somma di funzioni di classi \mathcal{C}^2, insieme chiuso rispetto a tali operazioni, dunque è essa stessa di classe \mathcal{C}^2. Inoltre, per semplificare l’esercizio, è utile sottolineare che la funzione presenta una periodicità in entrambe le variabili. Vale, infatti

        \begin{equation*} f(x+2j\pi,y+2k\pi)=f(x,y)\qquad\forall(x,y)\in\mathbb{R}^2,\qquad\forall j,k\in\mathbb{Z}. \end{equation*}

    Questo fatto permette di concentrare lo studio di f nella regione A\coloneqq[0,2\pi]\times[0,2\pi]\subset\mathbb{R}^2, poiché per periodicità possiamo estendere ogni considerazione a tutto il dominio di f. Restringiamo, allora, il problema di studiare i punti critici di f alla regione A. Per trovare i punti critici di f, calcoliamone le derivate parziali:

        \begin{equation*}     \dfrac{\partial f}{\partial x}(x,y)=\cos(x)\qquad\forall(x,y)\in A \end{equation*}

        \begin{equation*}     \dfrac{\partial f}{\partial y}(x,y)=\cos(y)\qquad\forall(x,y)\in A, \end{equation*}

    da cui

        \begin{equation*}     \nabla f(x,y)=(\cos(x),\cos(y))\qquad\forall(x,y)\in A. \end{equation*}

    I punti critici di f in A saranno le soluzioni (x,y)\in A dell’equazione \nabla f(x,y)=(0,0), ovvero i punti che soddisfano il seguente sistema:

        \begin{equation*}     \left\{\begin{array}{l}          \dfrac{\partial f}{\partial x}(x,y)=\cos(x)=0\\\\     \dfrac{\partial f}{\partial y}(x,y)=\cos(y)=0      \end{array}     \right.\Leftrightarrow     \left\{\begin{array}{l}     x=\dfrac{\pi}{2}+j\pi,\qquad j=0,1\\\\     y=\dfrac{\pi}{2}+k\pi,\qquad k=0,1     \end{array}     \right. \end{equation*}

    Abbiamo dunque trovato quattro punti critici di f su A:

        \begin{equation*} P_{0,0}=\left(\dfrac{\pi}{2},\dfrac{\pi}{2}\right),\qquad P_{1,0}=\left(\dfrac{3}{2}\pi,\dfrac{\pi}{2}\right),\qquad P_{0,1}=\left(\dfrac{\pi}{2},\dfrac{3}{2}\pi\right),\qquad P_{1,1}=\left(\dfrac{3}{2}\pi,\dfrac{3}{2}\pi\right). \end{equation*}

    Se volessimo considerare tutti i punti critici di f su \mathbb{R}^2 basta estendere per periodicità i punti critici trovati in A, ottenendo la famiglia di punti critici

        \[\left\{P_{j,k}\right\}_{j,k\in\mathbb{Z}}:=\left(\dfrac{\pi}{2}+j\pi,\dfrac{\pi}{2}+k\pi\right),\]

    parametrizzata da due parametri j,k\in\mathbb{Z}, rappresentata in figura 16.

        \[\quad\]

        \[\quad\]

        \[\quad\]

    Calcoliamo la matrice hessiana e valutiamola nei quattro punti per studiarne la natura. Prima, calcoliamo le derivate seconde di f:

        \begin{equation*}     \dfrac{\partial^2 f}{\partial x^2}(x,y)=-\sin(x)\qquad\forall(x,y)\in A, \end{equation*}

        \begin{equation*}     \dfrac{\partial^2 f}{\partial x \partial y}(x,y)=0\qquad\forall(x,y)\in A, \end{equation*}

        \begin{equation*}     \dfrac{\partial^2 f}{\partial y^2}(x,y)=-\sin(y)\qquad\forall(x,y)\in A, \end{equation*}

    dunque

        \begin{equation*}     \nabla^2f(x,y)=\begin{pmatrix}     -\sin(x)&0\\     0&-\sin(y)     \end{pmatrix}\qquad\forall(x,y)\in A. \end{equation*}

    Studiamo quindi \nabla^2f(P_{j,k}), per j,k=0,1.

        \[\quad\]

    • P_{0,0}. Si ha

          \begin{equation*}         \nabla^2f(P_{0,0})=\nabla^2f\left(\dfrac{\pi}{2},\dfrac{\pi}{2}\right)=\begin{pmatrix}         -1&0\\         0&-1         \end{pmatrix}.     \end{equation*}

      La matrice Hessiana è già in forma diagonale, per cui presenta già i suoi autovalori: \lambda_1=\lambda_2=-1. Avendo entrambi gli autovalori negativi, dalla proposizione 1.25, \nabla^2f(P_{0,0}) è definita negativa, per cui P_{0,0}=\left(\dfrac{\pi}{2},\dfrac{\pi}{2}\right) è un punto di massimo relativo per f. Il grafico di f in un intorno di P_{0,0} è rappresentato in figura 18.

          \[\quad\]

          \[\quad\]

      Figura 18: grafico di f in un intorno di P_{0,0}

          \[\quad\]

          \[\quad\]

    •  

    • P_{1,0}. Vale

          \begin{equation*}         \nabla^2f(P_{1,0})=\nabla^2f\left(\dfrac{3}{2}\pi,\dfrac{\pi}{2}\right)=\begin{pmatrix}         1&0\\         0&-1         \end{pmatrix}.     \end{equation*}

      La matrice Hessiana è di nuovo in forma diagonale, i suoi autovalori sono \lambda_1=1, \lambda_2=-1. Questa volta gli autovalori hanno segno opposto, per cui dalla proposizione 1.25, segue che \nabla^2f(P_{1,0}) è indefinita, dunque P_{1,0}=\left(\dfrac{3}{2}\pi,\dfrac{\pi}{2}\right) è un punto di sella per f, rappresentato in figura 19a.

    •  

    • P_{0,1}. Abbiamo

          \begin{equation*}         \nabla^2f(P_{0,1})=\nabla^2f\left(\dfrac{\pi}{2},\dfrac{3}{2}\pi\right)=\begin{pmatrix}         -1&0\\         0&1         \end{pmatrix}.     \end{equation*}

      i due autovalori hanno nuovamente segno opposto, dunque procedendo come per P_{1,0}, anche anche P_{0,1}=\left(\dfrac{\pi}{2},\dfrac{3}{2}\pi\right) è un punto di sella, rappresentato in figura 19a.

          \[\quad\]

          \[\quad\]

      Figura 19: grafici di f in intorni di alcuni dei punti studiati.

          \[\quad\]

          \[\quad\]

    •  

    • P_{1,1}. La matrice hessiana soddisfa

          \begin{equation*}         \nabla^2f(P_{1,1})=\nabla^2f\left(\dfrac{3}{2}\pi,\dfrac{3}{2}\pi\right)=\begin{pmatrix}         1&0\\         0&1         \end{pmatrix}.     \end{equation*}

      I due autovalori sono entrambi positivi, dunque \nabla^2f(P_{1,1}) è definita positiva e ciò significa che P_{1,1}=\left(\dfrac{3}{2}\pi,\dfrac{3}{2}\pi\right) è un punto di minimo relativo per f, rappresentato in figura 19b.

    Abbiamo dunque trovato che i punti P_{j,k} sono rispettivamente:

        \[\quad\]

    • di massimo relativo se j,k sono entrambi pari, rappresentati in verde in figura 20;
    •  

    • di minimo relativo se j,k sono entrambi dispari, rappresentati in rosso in figura 20;
    •  

    • di sella se j è pari e k dispari o viceversa, rappresentati in arancio in figura 20.

        \[\quad\]

        \[\quad\]

    Figura 20: in verde sono rappresentati i punti di massimo di f, in rosso quelli di minimo e in arancione i punti di sella.

        \[\quad\]

        \[\quad\]

    Osservazione. Notiamo che nel primo svolgimento l’unica regolarità richiesta per f è l’esistenza delle derivate parziali per determinare i punti critici, mentre nel secondo svolgimento è richiesta la regolarità \mathcal{C}^2 di f per applicare i risultati sulla definitezza della matrice hessiana.

    Inoltre, il primo svolgimento permette di dedurre che i punti di massimo e relativo sono anche rispettivamente di massimo e minimo assoluto, mentre tale conclusione non è immediatamente intuibile dal secondo svolgimento. Per ottenerla, occorrerebbe preliminarmente mostrare che punti di massimo e minimo assoluto esistono, ad esempio usando la periodicità di f e il teorema di Weierstrass, e successivamente confrontare i valori di f nei vari punti di massimo e minimo relativo per stabilire quale di essi è di massimo o minimo assoluto per f.


     
     

    Esercizio 3.2  (\bigstar\bigstar\largewhitestar\largewhitestar\largewhitestar). Sia data la seguente funzione f:\mathbb{R}^2\rightarrow\mathbb{R}, definita da

        \begin{equation*}     f(x,y):=(x-y)^4-8(x-y)^2\qquad\forall(x,y)\in\mathbb{R}^2. \end{equation*}

    Calcolare i punti critici di f e studiarne la natura.

    Svolgimento 1.

    Proponiamo una maniera rapida per risolvere l’esercizio che si applica per la natura della funzione da studiare. Osserviamo che f può essere espressa come funzione della sola variabile t\coloneqq x-y, dunque si possono studiare i punti critici della funzione g \colon \mathbb{R} \to \mathbb{R} definita da

        \[g(t)= t^4 - 8t^2\qquad\forall t\in\mathbb{R}.\]

    Vale infatti

        \[f(x,y)=g(x-y),\qquad\forall(x,y)\in\mathbb{R}^2,\]

    dunque

        \[\dfrac{\partial f}{\partial x}(x,y)=g'(x-y),\qquad \dfrac{\partial f}{\partial y}(x,y)=-g'(x-y)\]

    e quindi vale

        \[\nabla f(x,y)=(1,-1)g'(x-y)=0\iff g'(x-y)=0.\]

    Studiamo allora i punti critici di g:

        \[g'(t)=4t^3-16t=4t(t+2)(t-2)=0\iff t=0\lor t=\pm2.\]

    I valori t=0 e t=\pm2 corrispondono alle rette y=x e y=x\pm2. La natura di queste rette critiche dipende ancora da quella dei punti critici di g: studiamo il segno di g'' nei punti critici.

        \[g''(t)=12t^2-16,\]

    da cui

        \[g''(0)=-16<0,\qquad g''(2)=g''(-2)=32>0,\]

    dunque t=0 è un punto di massimo relativo per g, mentre t=\pm2 sono punti di minimo relativo. Ciò implica che la retta y=x sia costituita da punti di massimo relativo per f, mentre le rette y=x\pm2 da punti di minimo relativo.


    Svolgimento 2.

    Osserviamo che f è somma e prodotto di funzioni di classe \mathcal{C}^2, insieme chiuso rispetto a queste operazioni, pertanto f è essa stessa di classe \mathcal{C}^2 su \mathbb{R}^2. Calcoliamo le derivate parziali di f rispetto alle variabili x ed y:

        \[\begin{aligned}     \dfrac{\partial f}{\partial x}(x,y)&=4(x-y)^3-16(x-y)=4(x-y)((x-y)^2-4)\qquad\forall(x,y)\in\mathbb{R}^2;\\[4pt]     \dfrac{\partial f}{\partial y}(x,y)&=-4(x-y)^3+16(x-y)=-4(x-y)((x-y)^2-4)\qquad\forall(x,y)\in\mathbb{R}^2. \end{aligned}\]

    Dunque il gradiente di f risulta essere

        \[\begin{aligned}     \nabla f(x,y)&=(4(x-y)((x-y)^2-4),-4(x-y)((x-y)^2-4))\\     &=[4(x-y)((x-y)^2-4)](1,-1)\qquad\forall(x,y)\in\mathbb{R}^2. \end{aligned}\]

    I punti critici di f sono le coppie (x,y)\in\mathbb{R}^2 che annullano \nabla f, ovvero che risolvono l’equazione vettoriale

        \[\begin{aligned}     0=\nabla f(x,y)=[4(x-y)((x-y)^2-4)](1,-1)     \iff & 	4(x-y)((x-y)^2-4)=0 	\\     \iff &     x=y\ \lor \ y=x\pm 2. \end{aligned}\]

        \[\quad\]

        \[\quad\]

    Figura 21: punti critici di f, rappresentati dalle rette parallele r_k per k=-2,0,2.

        \[\quad\]

        \[\quad\]

    Abbiamo dunque trovato che il luogo geometrico dei punti critici di f corrisponde all’unione di tre rette parallele r_{-2},r_0 e r_2, di equazione rispettivamente y=x-2, y=x e y=x+2.

    Studiamo la natura di questi punti critici attraverso lo studio della matrice hessiana ristretta a queste rette. Possiamo già anticipare che la matrice hessiana sarà necessariamente semidefinita, in quanto f è costante nella direzione (1,1), quindi tale vettore annulla necessariamente la forma quadratica associata (teorema 1.13). Calcoliamo, per prima cosa, le derivate seconde di f:

        \[\begin{aligned}     \dfrac{\partial^2 f}{\partial x^2}(x,y)&=4((x-y)^2-4)+8(x-y)^2=4(3(x-y)^2-4)\qquad\forall(x,y)\in\mathbb{R}^2;\\[4pt]     \dfrac{\partial^2 f}{\partial x\partial y}(x,y)&=-4(3(x-y)^2-4)=-\dfrac{\partial^2 f}{\partial x^2}(x,y)\qquad\forall(x,y)\in\mathbb{R}^2;\\[4pt]     \dfrac{\partial^2 f}{\partial y^2}(x,y)&=4(3(x-y)^2-4)=\dfrac{\partial^2 f}{\partial x^2}(x,y)\qquad\forall(x,y)\in\mathbb{R}^2. \end{aligned}\]

    La matrice hessiana di f sarà dunque

    (9)   \begin{equation*} \nabla^2f(x,y) =\begin{pmatrix} 4(3(x-y)^2-4)&-4(3(x-y)^2-4)\\[5pt] -4(3(x-y)^2-4)&4(3(x-y)^2-4)     \end{pmatrix}     =4(3(x-y)^2-4)\begin{pmatrix}         1&-1\\         -1&1     \end{pmatrix}     \qquad\forall(x,y)\in\mathbb{R}^2. \end{equation*}

    Osserviamo immediatamente che la matrice hessiana di f è singolare in ogni punto. Infatti si ha

        \begin{equation*}     \det\nabla^2f(x,y)=[4(3(x-y)^2-4)]^2\det\begin{pmatrix}         1&-1\\         -1&1     \end{pmatrix}=0\qquad\forall(x,y)\in\mathbb{R}^2. \end{equation*}

    A causa della singolarità della matrice hessiana, per il teorema 1.17, essa non è conclusiva nel determinare la natura dei punti critici. Siamo, dunque, costretti ad affrontare il problema tramite altre vie.

    Osserviamo che f è costante sulle rette r_k:y=x+k, con k=-2,0,2, infatti per ogni x\in\mathbb{R}

    Mostriamo ora che

    (10)   \begin{equation*}     \operatorname{Im}(f) \subseteq     [-16,+\infty),  \end{equation*}

    e quindi che le rette r_{\pm2} di equazione y=x\pm2 sono costituite da punti di minimo assoluto (in particolare punti di minimo relativo) per f. Infatti è possibile esprimere f come differenza di due quadrati:

        \[\begin{aligned}     f(x,y)     &=(x-y)^4-8(x-y)^2     \\     &=(x-y)^4-8(x-y)^2+16-16     \\     &=[(x-y)^2+4]^2-16     \\     &\geq-16\qquad\forall(x,y)\in\mathbb{R}^2. \end{aligned}\]

    Dunque tutti i punti della forma \{(x,x\pm 2)\mid x\in\mathbb{R}\}, appartenti cioè ad r_{\pm2} sono punti di minimo assoluto per f.

    Figura 22: la funzione f rappresentata in un intorno di r_{2}. La situazione in r_{-2} è analoga.

    Per quanto riguarda la retta r_0, di equazione y=x, consideriamo l’intorno tubolare di r_0 di raggio 1, rappresentato in verde in figura 23 e definito da

    (11)   \begin{equation*}     \mathcal{U}_{r_0}:=\{(x,y)\in\mathbb{R}^2\mid |x-y|<1\}. \end{equation*}

        \[\quad\]

        \[\quad\]

        \[\quad\]

    Studiamo f in tale intorno:

        \begin{equation*}     f|_{\mathcal{U}_{r_0}}(x,y)=(x-y)^4-8(x-y)^2=(x-y)^2[(x-y)^2-8]\leq 0,\qquad\forall(x,y)\in\mathcal{U}_{r_0} \end{equation*}

    poiché in \mathcal{U}_{r_0}, |x-y|<1. Considerando il fatto, già esplicitato, che f si annulla sulla retta r_0, possiamo affermare che i punti critici appartenenti alla retta r_0 sono punti di massimo relativo, come visibile anche dal grafico di f in figura 24.


     
     

    Esercizio 3.3  (\bigstar\bigstar\bigstar\largewhitestar\largewhitestar). Determinare i punti critici su \mathbb{R}^2 delle funzioni f_k \colon \mathbb{R}^2 \to \mathbb{R} definite da

        \begin{equation*}     f_k(x,y):=x^2+2kxy+y^2, \qquad\forall(x,y)\in\mathbb{R}^2, \end{equation*}

    al variare del parametro k\in\mathbb{R} e stabilire se essi sono di massimo, minimo o sella.

    Svolgimento.

    Osserviamo che f_k è di classe \mathcal{C}^2 su \mathbb{R}^2 per ogni k\in\mathbb{R}. Troviamo allora i punti critici di f_k, calcolando le sue derivate prime

        \begin{equation*}     \dfrac{\partial f_k}{\partial x}(x,y)=2x+2ky,\qquad     \dfrac{\partial f_k}{\partial y}(x,y)=2kx+2y,\qquad\forall(x,y)\in\mathbb{R}^2. \end{equation*}

    Determiniamo i punti critici, ovvero le soluzioni del sistema

        \begin{equation*}     \left\{\begin{array}{l}          \dfrac{\partial f_k}{\partial x}(x,y)=2x+2ky=0  \\\\          \dfrac{\partial f_k}{\partial y}(x,y)=2kx+2y=0      \end{array}\right.\Leftrightarrow      \left\{\begin{array}{l}          x=-ky  \\\\          y+kx=0     \end{array}\right.\Leftrightarrow\left\{\begin{array}{l}          x=-ky  \\\\          y(1-k^2)=0.     \end{array}\right. \end{equation*}

    Se k\neq\pm1, la seconda equazione ha soluzione solo se y=0, che implica x=0, trovando come unico punto critico di f_k il punto P_1=(0,0).

    Supponiamo invece k=\pm1, per cui la seconda equazione è automaticamente soddisfatta. In entrambi i casi avremo una retta interamente composta da punti critici: i punti critici di f_1 sono sulla retta y=-x, mentre quelli di f_{-1} sulla retta y=x.

        \[\quad\]

        \[\quad\]

        \[\quad\]

        \[\quad\]

        \[\quad\]

    Per quanto riguarda lo studio della natura dei punti critici proponiamo due diversi procedimenti.

    Procedimento 1. Consideriamo separatamente i casi |k|<1, |k|>1 e |k|=1.

        \[\quad\]

    • |k|<1. Sviluppando i quadrati (x+y)^2 e (x-y)^2, si ottiene la disuguaglianza x^2+y^2 \geq 2|x||y| con l’uguaglianza che vale se e solo se |x|=|y|, da cui segue

      (12)   \begin{equation*} f_k(x,y) = x^2 + 2k xy + y^2 = x^2 + y^2 - 2|x||y| + (2|x||y| + 2kxy) > 0 \qquad \forall (x,y) \in \mathbb{R}^2 \setminus \{(0,0\}). \end{equation*}

      Ciò, insieme al fatto che f(0,0)=0, implica che (0,0) è un punto di minimo assoluto per f_k.

    •  

    • |k|>1. Considerando le restrizioni di f_k alle rette di equazioni x=y e x=-y si ottiene

      (13)   \begin{equation*} \begin{gathered} f_k(x,x)= 2(k+1)x^2  \begin{cases} >0  	& \forall x \neq 0, \text{ se } k>0\\ <0  	& \forall x \neq 0, \text{ se } k<0, \end{cases} \\[4pt] f_k(x,-x)= 2(-k+1)x^2  \begin{cases} <0  	& \forall x \neq 0, \text{ se } k>0\\ >0  	& \forall x \neq 0, \text{ se } k<0. \end{cases}  \end{gathered} \end{equation*}

      Da ciò si evince che f_k assume valori di segno opposto in ogni intorno di (0,0), che risulta quindi un punto di sella per f_k.

    •  

    • |k|=1. Se k=\pm 1, si ha

      (14)   \begin{equation*} f_k(x,y)= x^2 \pm 2 xy + y^2 = (x \pm y)^2 \geq 0 \qquad \forall (x,y) \in \mathbb{R}^2, \end{equation*}

      con l’uguaglianza valida sulle rette x = \mp y, che risultano quindi costituite da punti di minimo assoluto per f_k.

      Procedimento 2. Calcoliamo le derivate seconde di f_k per costruire la sua matrice hessiana:

          \begin{equation*}     \dfrac{\partial^2f_k}{\partial x^2}(x,y)=2,\qquad\dfrac{\partial^2f_k}{\partial x\partial y}(x,y)=2k,\qquad\dfrac{\partial^2f_k}{\partial y^2}(x,y)=2, \end{equation*}

      dunque la matrice hessiana di f_k risulta essere

          \begin{equation*}     \nabla^2f_k(x,y)=\begin{pmatrix}     2&2k\\     2k&2     \end{pmatrix}\qquad\forall k \in \mathbb{R}, \,\, \forall (x,y) \in \mathbb{R}^2. \end{equation*}

      Notando che \nabla^2f_k è costante, le considerazioni future saranno valide per ogni punto critico. Per determinarne la segnatura possiamo sia calcolare esplicitamente i suoi autovalori, sia sfruttare il criterio di Sylvester 1.28.

          \[\quad\]

      1. Iniziamo con il primo metodo. Gli autovalori sono radici del polinomio caratteristico \det(\nabla^2f_k-\lambda \operatorname{Id}), che indicheremo con p_{\nabla^2f_k}(\lambda):

            \begin{equation*}     p_{\nabla^2f_k}(\lambda)=\det\begin{pmatrix}     2-\lambda&2k\\     2k&2-\lambda     \end{pmatrix}=(2-\lambda)^2-4k^2=(2-\lambda+2k)(2-\lambda-2k), \end{equation*}

        le cui soluzioni sono

            \begin{equation*}     \lambda_{1,2}=2\pm2k. \end{equation*}

        Ricordiamo che la definitezza della matrice dipende dal segno dei suoi autovalori. Studiamo quindi il segno di \lambda_1 e \lambda_2 al variare di k\in\mathbb{R}.

            \begin{equation*}     \begin{array}{l}          \lambda_1>0\iff k>-1 \\\\           \lambda_2>0\iff k<1     \end{array} \end{equation*}

        Ciò significa che

            \[\quad\]

        • \lambda_1,\lambda_2>0 per k \in (-1,1), \implies \nabla^2f_k definita positiva;
        •  

        • \lambda_1 e \lambda_2 hanno segno opposto per k\in(-\infty,-1)\cup(1,+\infty), \implies \nabla^2f_k indefinita;
        •  

        • \lambda_1=0 o \lambda_2=0 per k=\pm1, \implies \nabla^2f_k semidefinita positiva, ma non definita positiva.

        La situazione è rappresentata in figura 28.

            \[\quad\]

            \[\quad\]

        Rendered by QuickLaTeX.com

        Figura 28: segnatura della matrice hessiana \nabla^2 f_k al variare di k \in \mathbb{R}; in verde l’intervallo in cui \nabla^2 f_k è definita positiva, in blu gli intervalli per cui \nabla^2 f_k è indefinita, in rosso valori di k per cui \nabla^2 f_k è semidefinita positiva, ma non definita.

            \[\quad\]

            \[\quad\]

      2.  

      3. Procediamo ora con il criterio di Sylvester 1.28, il quale consiste nello studio del segno dei minori nord-ovest e di quelli principali per determinare la definitezza della matrice. Calcoliamo i minori nord-ovest:

            \begin{equation*}     d_1=\det(2)=2>0,\qquad d_2=\det\begin{pmatrix}     2&2k\\     2k&2     \end{pmatrix}=4-4k^2. \end{equation*}

        Il primo minore non dipende da k ed è sempre positivo, dunque possiamo escludere che la matrice sia semidefinita negativa e tantomeno definita negativa. Studiamo il segno del secondo determinante: possiamo distinguere nuovamente tre casi:

            \[\quad\]

        • d_2>0 per -1<k<1, \implies \nabla^2f_k definita positiva;
        •  

        • d_2<0 per k\in(-\infty,-1)\cup(1,+\infty), \implies \nabla^2f_k indefinita;
        •  

        • d_2=0 per k=\pm1, \implies \nabla^2f_k semidefinita positiva, ma non definita positiva.

        Ovviamente, entrambi i metodi portano alla stessa conclusione. Sfruttiamo quindi lo studio della matrice hessiana per determinare la natura dei punti critici. Per k\neq\pm1 avevamo trovato il solo punto critico P_1=(0,0). Dallo studio precedente, applicando il teorema 1.17, risulta che

        – se -1<k<1, \nabla^2f_k(P_1) sia definita positiva \implies P_1 è un punto di minimo relativo per f_k, come rappresentato in figura 29.

      4. – se k\in(-\infty,-1)\cup(1,+\infty), \nabla^2f_k(P_1) è indefinita \implies P_1 è un punto di sella per f_k, come si vede nella figura 30.

            \[\quad\]

            \[\quad\]

            \[\quad\]

         

      5. Se k=\pm1, abbiamo la retta di punti critici y=\mp x per f_{\pm1}. Tuttavia, in tutti questi punti la matrice hessiana risulta semidefinita positiva, ma non definita positiva. Ancora dal teorema 1.17 deduciamo che i punti della retta possono essere punti di minimo relativi o punti di sella.

        Occorre quindi determinare in altro modo la natura dei punti critici della retta y=x per f_{-1} e quella di y=-x per f_1. Osserviamo che

            \begin{equation*}     f_{-1}(x,y)=x^2-2xy+y^2=(x-y)^2\qquad\forall(x,y)\in\mathbb{R}^2. \end{equation*}

        Ne deduciamo che f_{-1}\geq0 su \mathbb{R}^2 e i punti sulla retta y=x soddisfano f_{-1}(x,y)=0, dunque per definizione la retta y=x è un luogo di punti di minimo relativo (anche assoluto) per f_{-1}, come si vede nella figura 31.

            \[\quad\]

            \[\quad\]

            \[\quad\]

        Considerazioni analoghe valgono per i punti sulla retta y=-x per f_1, infatti

            \begin{equation*}     f_{1}(x,y)=x^2+2xy+y^2=(x+y)^2\geq0\qquad\forall(x,y)\in\mathbb{R}^2 \end{equation*}

        e se y=-x, f(x,y)=0. Dunque anche i punti della retta y=-x sono punti di minimo relativo (anche assoluto) per f_1, come evidente dalla figura 32.


     
     

    Esercizio 3.4  (\bigstar\largewhitestar\largewhitestar\largewhitestar\largewhitestar). Determinare i punti critici su \mathbb{R}^3 della funzione f:\mathbb{R}^3\rightarrow\mathbb{R}, definita da

        \begin{equation*}     f(x,y,z):=(x^2+y^2)^2-xy+z^2,\qquad\forall(x,y,z)\in\mathbb{R}^3 \end{equation*}

    e stabilire se essi sono di massimo, minimo o sella.

    Svolgimento.

    Osserviamo che f è di classe \mathcal{C}^2 su \mathbb{R}^3. Andiamo alla ricerca dei punti critici di f, calcolando le sue derivate prime. Per ogni (x,y,z)\in\mathbb{R}^3 si ha

        \begin{equation*}     \dfrac{\partial f}{\partial x}(x,y,z)=4x(x^2+y^2)-y,\qquad     \dfrac{\partial f}{\partial y}(x,y,z)=4y(x^2+y^2)-x,\qquad  \dfrac{\partial f}{\partial y}(x,y,z)=2z \end{equation*}

    e determiniamo i punti critici, ovvero le soluzioni del sistema

        \begin{equation*} \begin{cases}          \dfrac{\partial f}{\partial x}(x,y,z)=4x(x^2+y^2)-y=0  \\[8pt]          \dfrac{\partial f}{\partial y}(x,y,z)=4y(x^2+y^2)-x=0 \\[8pt]          \dfrac{\partial f}{\partial z}(x,y,z)=2z=0     \end{cases}     \iff \begin{cases}          y=4x(x^2+y^2)  \\[4pt]          x=4y(x^2+y^2)\\[4pt]          z=0.     \end{cases} \end{equation*}

    Per studiare le prime due equazioni del sistema, sarebbe opportuno moltiplicare le equazioni per x e per y rispettivamente. Per farlo, tuttavia, dovremo supporre x,y\neq0. Pertanto, prima di procedere, dobbiamo controllare che le condizioni x=0 e y=0 non portino soluzioni al sistema. Si vede immediatamente che supponendo x=0 risulta y=0 e viceversa. Pertanto P_1:=(0,0,0) è il primo punto critico di f e l’unico per il quale x=0 o y=0.

    Ora è del tutto lecito supporre x,y\neq0 e continuiamo a risolvere il sistema moltiplicando la prima equazione per x e la seconda per y:

        \begin{equation*}     \left\{\begin{array}{l}          xy=4x^2(x^2+y^2)  \\[4pt]          xy=4y^2(x^2+y^2)\\[4pt]          z=0     \end{array}\right.\Leftrightarrow     \left\{\begin{array}{l}          4y^2(x^2+y^2)=4x^2(x^2+y^2)  \\[4pt]          xy=4y^2(x^2+y^2)\\[4pt]          z=0.     \end{array}\right. \end{equation*}

    Nella prima equazione, il termine x^2+y^2 è sicuramente positivo nelle nostre ipotesi, pertanto semplificabile. Lo stesso vale per y nella seconda equazione. Il sistema si riduce quindi a

        \begin{equation*}     \left\{\begin{array}{l}          y^2=x^2  \\[4pt]          x=4y(x^2+y^2)\\[4pt]          z=0     \end{array}\right.\Leftrightarrow     \left\{\begin{array}{l}          y=\pm x  \\[4pt]          x=4y(x^2+y^2)\\[4pt]          z=0.     \end{array}\right. \end{equation*}

    Inseriamo la prima equazione nella seconda, ottenendo

        \begin{equation*}     x=\pm8x^3\iff x^2=\pm\dfrac{1}{8}. \end{equation*}

    L’equazione x^2=-\dfrac{1}{8} non ha chiaramente soluzione, dunque la condizione y=-x non comporta alcun punto critico. Invece, per y=x l’equazione risulta

        \begin{equation*}     x^2=\dfrac{1}{8}\iff x=\pm\dfrac{\sqrt{2}}{4}, \end{equation*}

    da cui

        \begin{equation*}     y=\pm\dfrac{\sqrt{2}}{4}. \end{equation*}

    Abbiamo dunque trovato altri due punti critici di f:

        \begin{equation*}     P_2=\left(\dfrac{\sqrt{2}}{4},\dfrac{\sqrt{2}}{4},0\right),\qquad P_3=\left(-\dfrac{\sqrt{2}}{4},-\dfrac{\sqrt{2}}{4},0\right). \end{equation*}

    I punti P_1, P_2 e P_3 sono rappresentati in figura 33.

        \[\quad\]

        \[\quad\]

    Figura 33: i punti critici di f appartengono al piano z=0.

        \[\quad\]

        \[\quad\]

    Calcoliamo le derivate seconde di f per costruire la matrice hessiana di f:

        \begin{equation*}     \begin{array}{lll}          &\dfrac{\partial^2f}{\partial x^2}(x,y,z)=4(x^2+y^2)+8x^2=4(3x^2+y^2)\qquad\forall(x,y,z)\in\mathbb{R}^3,\\[8pt]          &\dfrac{\partial^2f}{\partial x\partial y}(x,y,z)=8xy-1\qquad\forall(x,y,z)\in\mathbb{R}^3,\\[8pt]          &\dfrac{\partial^2f}{\partial x\partial z}(x,y,z)=0\qquad\forall(x,y,z)\in\mathbb{R}^3,\\[8pt]          &\dfrac{\partial^2f}{\partial y^2}(x,y)=4(x^2+y^2)+8y^2=4(x^2+3y^2)\qquad\forall(x,y,z)\in\mathbb{R}^3,\\[8pt]          &\dfrac{\partial^2f}{\partial y\partial z}(x,y,z)=0\qquad\forall(x,y,z)\in\mathbb{R}^3,\\[8pt]          &\dfrac{\partial^2f}{\partial^2 z}(x,y,z)=2\qquad\forall(x,y,z)\in\mathbb{R}^3,\\[8pt]           \end{array}         \end{equation*}

    dunque la matrice hessiana di f risulta essere

        \begin{equation*}     \nabla^2f(x,y,z)=\begin{pmatrix}     4(3x^2+y^2)&8xy-1&0\\[3pt]     8xy-1&4(x^2+3y^2)&0\\[3pt]     0&0&2     \end{pmatrix}\qquad\forall(x,y,z)\in\mathbb{R}^3. \end{equation*}

    Valutiamo la matrice hessiana di f in ciascuno dei punti critici di f precedentemente trovati:

        \begin{equation*}     \nabla^2f(P_1)=\begin{pmatrix}     0&-1&0\\[3pt]     -1&0&0\\[3pt]     0&0&2     \end{pmatrix},\qquad\nabla^2f(P_2)=\nabla^2f(P_3)=\begin{pmatrix}     2&0&0\\[3pt]     0&2&0\\[3pt]     0&0&2     \end{pmatrix}. \end{equation*}

    Studiamo la definitezza di tali matrici.

    P_1) Possiamo procedere in entrambi i modi: sia calcolando esplicitamente il segno degli autovalori di \nabla^2f(P_1), sia attraverso il criterio di Sylvester 1.28.

        \[\quad\]

    • Detta M_1\coloneqq\nabla^2f(P_1), troviamo gli autovalori di M_1, ovvero le radici del polinomio caratteristico:

          \begin{equation*} \begin{split}             p_{M_1}(\lambda)             \coloneqq &             \det(M_1-\lambda \operatorname{Id})             \\[3pt]             =&             \det\begin{pmatrix}             -\lambda&-1&0\\[3pt]     -1&-\lambda&0\\[3pt]     0&0&2-\lambda             \end{pmatrix}             \\[3pt]             =&             (2-\lambda)(\lambda^2-1)             \\             =&             (\lambda+1)(\lambda-1)(2-\lambda)=0. \end{split}         \end{equation*}

      Le soluzioni di questa equazione sono

          \begin{equation*}              \lambda_1=1,\qquad\lambda_2=-1,\qquad\lambda_3=2.         \end{equation*}

      È immediato notare che gli autovalori di \nabla^2f(P_1) hanno segno discorde, pertanto \nabla^2g(P_1) è indefinita.

    •  

    • Avremmo potuto anche applicare il criterio di Sylvester 1.28, osservando che il minore nord-ovest di ordine due è negativo:

          \begin{equation*}         d_2=\det\begin{pmatrix}     0&-1\\[3pt]     -1&0     \end{pmatrix}=-1.     \end{equation*}

      Ciò è sufficiente, grazie al teorema 1.28, per concludere che \nabla^2f(P_1) è indefinita.

    P_2,P_3) Osserviamo che la matrice \nabla^2f(P_2)=\nabla^2f(P_3) è già diagonale

        \begin{equation*}         \nabla^2f(P_2)=\nabla^2f(P_3)=\begin{pmatrix}     2&0&0\\[3pt]     0&2&0\\[3pt]     0&0&2     \end{pmatrix}     \end{equation*}

    e gli elementi che compaiono sulla diagonale sono (per definizione) esattamente gli autovalori della matrice. Segue immediatamente che i suoi autovalori hanno tutti segno positivo, pertanto la matrice è definita positiva.

    Grazie allo studio della definitezza delle matrici hessiane valutate in ciascun punto critico possiamo determinare la loro natura, applicando il teorema 1.17: \nabla^2f(P_1) è indefinita, dunque P_1 è un punto di sella per f, mentre in P_2 e P_3, \nabla^2f(P_2)=\nabla^2f(P_3) è definita positiva e quindi P_2 e P_3 sono punti di minimo relativo per f.

    Osserviamo che, essendo f è di classe \mathcal{C}^1 su \mathbb{R}^3 e che \mathbb{R}^3 è aperto, il teorema di Fermat 1.7 garantisce che P_2 e P_3 sono gli unici estremali relativi di f.


     
     

    Riferimenti bibliografici

    [1] Bramanti, M. & Pagani C. D. & Salsa S., Analisi Matematica 2, Zanichelli (2016).

     
     

    Tutta la teoria di analisi matematica

    Leggi...

    1. Teoria Insiemi
    2. Il metodo della diagonale di Cantor
    3. Logica elementare
    4. Densità dei numeri razionali nei numeri reali
    5. Insiemi Numerici \left(\mathbb{N},\, \mathbb{Z},\, \mathbb{Q}\right)
    6. Il principio di induzione
    7. Gli assiomi di Peano
    8. L’insieme dei numeri reali: costruzione e applicazioni
    9. Concetti Fondamentali della Retta Reale: Sintesi Teorica
    10. Costruzioni alternative di \mathbb{R}
    11. Binomio di Newton
    12. Spazi metrici, un’introduzione
    13. Disuguaglianza di Bernoulli
    14. Disuguaglianza triangolare
    15. Teoria sulle funzioni
    16. Funzioni elementari: algebriche, esponenziali e logaritmiche
    17. Funzioni elementari: trigonometriche e iperboliche
    18. Funzioni goniometriche: la guida essenziale
    19. Teorema di Bolzano-Weierstrass per le successioni
    20. Criterio del rapporto per le successioni
    21. Definizione e proprietà del numero di Nepero
    22. Limite di una successione monotona
    23. Successioni di Cauchy
    24. Il teorema ponte
    25. Teoria sui limiti
    26. Simboli di Landau
    27. Funzioni continue – Teoria
    28. Il teorema di Weierstrass
    29. Il teorema dei valori intermedi
    30. Il teorema della permanenza del segno
    31. Il teorema di Heine-Cantor
    32. Il teorema di esistenza degli zeri
    33. Il metodo di bisezione
    34. Teorema ponte versione per le funzioni continue
    35. Discontinuità di funzioni monotone
    36. Continuità della funzione inversa
    37. Teorema delle contrazioni o Teorema di punto fisso di Banach-Caccioppoli
    38. Teoria sulle derivate
    39. Calcolo delle derivate: la guida pratica
    40. Teoria sulle funzioni convesse
    41. Il teorema di Darboux
    42. I teoremi di de l’Hôpital
    43. Teorema di Fermat
    44. Teoremi di Rolle e Lagrange
    45. Il teorema di Cauchy
    46. Espansione di Taylor: teoria, esempi e applicazioni pratiche
    47. Polinomi di Taylor nei limiti: istruzioni per l’uso
    48. Integrali definiti e indefiniti
    49. Teorema fondamentale del calcolo integrale (approfondimento)
    50. Integrali ricorsivi
    51. Formule del trapezio, rettangolo e Cavalieri-Simpson
    52. Teoria sugli integrali impropri
    53. Funzioni integrali – Teoria
    54. Introduzione ai numeri complessi – Volume 1 (per un corso di ingegneria — versione semplificata)
    55. Introduzione ai numeri complessi – Volume 1 (per un corso di matematica o fisica)
    56. Serie numeriche: la guida completa
    57. Successioni di funzioni – Teoria
    58. Teoremi sulle successioni di funzioni
      1. 58a. Criterio di Cauchy per la convergenza uniforme
      2. 58b. Limite uniforme di funzioni continue
      3. 58c. Passaggio al limite sotto il segno di integrale
      4. 58d. Limite uniforme di funzioni derivabili
      5. 58e. Piccolo teorema del Dini
      6. 58f. Procedura diagonale e teorema di Ascoli-Arzela
    59. Serie di funzioni – Teoria
    60. Serie di potenze – Teoria
    61. Serie di Fourier – Teoria e applicazioni
    62. Integrali multipli — Parte 1 (teoria)
    63. Integrali multipli — Parte 2 (teoria e esercizi misti)
    64. Regola della Catena — Teoria ed esempi.
    65. Jacobiano associato al cambiamento di coordinate sferiche
    66. Guida ai Massimi e Minimi: Tecniche e Teoria nelle Funzioni Multivariabili
    67. Operatore di Laplace o Laplaciano
    68. Teoria equazioni differenziali
    69. Equazione di Eulero
    70. Teoria ed esercizi sulla funzione Gamma di Eulero
    71. Teoria ed esercizi sulla funzione Beta
    72. Approfondimento numeri complessi
    73. Diverse formulazioni dell’assioma di completezza
    74. Numeri di Delannoy centrali
    75. Esercizi avanzati analisi

     
     

    Tutte le cartelle di Analisi Matematica

    Leggi...

    1. Prerequisiti di Analisi
      1. Ripasso algebra biennio liceo
      2. Ripasso geometria analitica
      3. Ripasso goniometria e trigonometria
      4. Errori tipici da evitare
      5. Insiemi numerici N,Z,Q,R
      6. Funzioni elementari
      7. Logica elementare
      8. Insiemi
    2. Successioni
      1. Teoria sulle Successioni
      2. Estremo superiore e inferiore
      3. Limiti base
      4. Forme indeterminate
      5. Limiti notevoli
      6. Esercizi misti Successioni
      7. Successioni per ricorrenza
    3. Funzioni
      1. Teoria sulle funzioni
      2. Verifica del limite in funzioni
      3. Limite base in funzioni
      4. Forme indeterminate in funzioni
      5. Limiti notevoli in funzioni
      6. Calcolo asintoti
      7. Studio di funzione senza derivate
      8. Dominio di una funzione
      9. Esercizi misti Funzioni
      10. Esercizi misti sui Limiti
    4. Funzioni continue-lipschitziane-holderiane
      1. Teoria sulle Funzioni continue-lipschitziane-holderiane
      2. Continuità delle funzioni
      3. Continuità uniforme
      4. Teorema degli zeri
      5. Esercizi sul teorema di Weierstrass senza l’uso delle derivate
    5. Calcolo differenziale
      1. Derivate
      2. Calcolo delle derivate
      3. Retta tangente nel calcolo differenziale
      4. Punti di non derivabilità nel calcolo differenziale
      5. Esercizi sul teorema di Weierstrass con l’uso delle derivate
      6. Studio di funzione completo nel calcolo differenziale
      7. Esercizi teorici nel calcolo differenziale
      8. Metodo di bisezione
      9. Metodo di Newton
    6. Teoremi del calcolo differenziale
      1. Teoria sui Teoremi del calcolo differenziale
      2. Teorema di Rolle
      3. Teorema di Lagrange
      4. Teorema di Cauchy
      5. Teorema di De L’Hôpital
    7. Calcolo integrale
      1. Integrale di Riemann
      2. Integrali immediati
      3. Integrale di funzione composta
      4. Integrali per sostituzione
      5. Integrali per parti
      6. Integrali di funzione razionale
      7. Calcolo delle aree
      8. Metodo dei rettangoli e dei trapezi
      9. Esercizi Misti Integrali Indefiniti
      10. Esercizi Misti Integrali Definiti
    8. Integrali impropri
      1. Teoria Integrali impropri
      2. Carattere di un integrale improprio
      3. Calcolo di un integrale improprio
    9. Espansione di Taylor
      1. Teoria Espansione di Taylor
      2. Limiti di funzione con Taylor
      3. Limiti di successione con Taylor
      4. Stime del resto
    10. Funzioni integrali (Approfondimento)
      1. Teoria Funzioni integrali (Approfondimento)
      2. Studio di funzione integrale
      3. Limiti con Taylor e De L’Hôpital
      4. Derivazione di integrali parametrici (Tecnica di Feynmann)
    11. Numeri Complessi
      1. Teoria Numeri complessi
      2. Espressioni con i numeri complessi
      3. Radice di un numero complesso
      4. Equazioni con i numeri complessi
      5. Disequazioni con i numeri complessi
      6. Esercizi misti Numeri complessi
    12. Serie numeriche
      1. Teoria Serie numeriche
      2. Esercizi Serie a termini positivi
      3. Esercizi Serie a termini di segno variabile
      4. Esercizi Serie geometriche e telescopiche
    13. Successioni di funzioni
      1. Teoria Successioni di funzioni
      2. Esercizi Successioni di funzioni
    14. Serie di funzioni
      1. Teoria Serie di funzioni
      2. Esercizi Serie di funzioni
    15. Serie di potenze
      1. Teoria Serie di potenze
      2. Esercizi Serie di potenze
    16. Serie di Fourier
      1. Teoria Serie di Fourier
      2. Esercizi Serie di Fourier
    17. Trasformata di Fourier
      1. Teoria Trasformata di Fourier
      2. Esercizi Trasformata di Fourier
    18. Funzioni di più variabili
      1. Teoria Funzioni di più variabili
      2. Massimi e minimi liberi e vincolati
      3. Limiti in due variabili
      4. Integrali doppi
      5. Integrali tripli
      6. Integrali di linea di prima specie
      7. Integrali di linea di seconda specie
      8. Forme differenziali e campi vettoriali
      9. Teorema di Gauss-Green
      10. Integrali di superficie
      11. Flusso di un campo vettoriale
      12. Teorema di Stokes
      13. Teorema della divergenza
      14. Campi solenoidali
      15. Teorema del Dini
    19. Equazioni differenziali lineari e non lineari
      1. Teoria equazioni differenziali lineari e non lineari
      2. Equazioni differenziali lineari e non lineari del primo ordine omogenee
    20. Equazioni differenziali lineari
      1. Del primo ordine non omogenee
      2. Di ordine superiore al primo,a coefficienti costanti,omogenee
      3. Di ordine superiore al primo,a coefficienti costanti,non omogenee
      4. Di Eulero,di Bernoulli,di Clairaut,di Lagrange e di Abel
      5. Non omogenee avente per omogenea associata un’equazione di Eulero
      6. Sistemi di EDO
    21. Equazioni differenziali non lineari
      1. A variabili separabiliO
      2. A secondo membro omogeneo
      3. Del tipo y’=y(ax+by+c)
      4. Del tipo y’=y(ax+by+c)/(a’x+b’y+c’)
      5. Equazioni differenziali esatte
      6. Mancanti delle variabili x e y
      7. Cenni sullo studio di un’assegnata equazione differenziale non lineare
      8. Di Riccati
      9. Cambi di variabile: simmetrie di Lie
    22. Analisi complessa
      1. Fondamenti
      2. Funzioni olomorfe
      3. Integrale di Cauchy e applicazioni
      4. Teorema della curva di Jordan e teorema fondamentale dell’Algebra
      5. Teorema di inversione di Lagrange
      6. Teorema dei Residui
      7. Funzioni meromorfe
      8. Prodotti infiniti e prodotti di Weierstrass
      9. Continuazione analitica e topologia
      10. Teoremi di rigidità di funzioni olomorfe
      11. Trasformata di Mellin
    23. Equazioni alle derivate parziali
      1. Equazioni del primo ordine
      2. Equazioni del secondo ordine lineari
      3. Equazioni non-lineari
      4. Sistemi di PDE
    24. Funzioni speciali
      1. Funzione Gamma di Eulero
      2. Funzioni Beta,Digamma,Trigamma
      3. Integrali ellittici
      4. Funzioni di Bessel
      5. Funzione zeta di Riemann e funzioni L di Dirichlet
      6. Funzione polilogaritmo
      7. Funzioni ipergeometriche
    25. Analisi funzionale
      1. Misura e integrale di Lebesgue
      2. Spazi Lp,teoremi di completezza e compattezza
      3. Spazi di Hilbert,serie e trasformata di Fourier
      4. Teoria e pratica dei polinomi ortogonali
      5. Spazi di Sobolev
    26. Complementi
      1. Curiosità e approfondimenti
      2. Compiti di analisi
      3. Esercizi avanzati analisi
    27. Funzioni Convesse

     
     

    Tutti gli esercizi di geometria

    In questa sezione vengono raccolti molti altri esercizi che coprono tutti gli argomenti di geometria proposti all’interno del sito con lo scopo di offrire al lettore la possibilità di approfondire e rinforzare le proprie competenze inerenti a tali argomenti.

    Strutture algebriche.





     
     

    Risorse didattiche aggiuntive per approfondire la matematica

    Leggi...

    • Math Stack Exchange – Parte della rete Stack Exchange, questo sito è un forum di domande e risposte specificamente dedicato alla matematica. È una delle piattaforme più popolari per discutere e risolvere problemi matematici di vario livello, dall’elementare all’avanzato.
    • Art of Problem Solving (AoPS) – Questo sito è molto noto tra gli studenti di matematica di livello avanzato e i partecipanti a competizioni matematiche. Offre forum, corsi online, e risorse educative su una vasta gamma di argomenti.
    • MathOverflow – Questo sito è destinato a matematici professionisti e ricercatori. È una piattaforma per domande di ricerca avanzata in matematica. È strettamente legato a Math Stack Exchange ma è orientato a un pubblico con una formazione più avanzata.
    • PlanetMath – Una comunità collaborativa di matematici che crea e cura articoli enciclopedici e altre risorse di matematica. È simile a Wikipedia, ma focalizzata esclusivamente sulla matematica.
    • Wolfram MathWorld – Una delle risorse online più complete per la matematica. Contiene migliaia di articoli su argomenti di matematica, creati e curati da esperti. Sebbene non sia un forum, è una risorsa eccellente per la teoria matematica.
    • The Math Forum – Un sito storico che offre un’ampia gamma di risorse, inclusi forum di discussione, articoli e risorse educative. Sebbene alcune parti del sito siano state integrate con altri servizi, come NCTM, rimane una risorsa preziosa per la comunità educativa.
    • Stack Overflow (sezione matematica) – Sebbene Stack Overflow sia principalmente noto per la programmazione, ci sono anche discussioni rilevanti di matematica applicata, specialmente nel contesto della scienza dei dati, statistica, e algoritmi.
    • Reddit (r/Math) – Un subreddit popolare dove si possono trovare discussioni su una vasta gamma di argomenti matematici. È meno formale rispetto ai siti di domande e risposte come Math Stack Exchange, ma ha una comunità attiva e molte discussioni interessanti.
    • Brilliant.org – Offre corsi interattivi e problemi di matematica e scienza. È particolarmente utile per chi vuole allenare le proprie capacità di problem solving in matematica.
    • Khan Academy – Una risorsa educativa globale con lezioni video, esercizi interattivi e articoli su una vasta gamma di argomenti di matematica, dalla scuola elementare all’università.






    Document









    Document